Você está na página 1de 82

I lecture

\\\\ In adipose tissue energy is stored in the form of: \\\ Glycogen \\ Triacylglycerols \\\ Proteins \\\ ATP

zaur ismailov

\\\\ Which of the following are the major regulatory hormones of carbohydrate metabolism? \\\ glucocorticoids and mineralocorticoids \\ Insulin and glucagon \\\ Thyroid and parathyroid hormones \\\ Growth hormone and prolactin \\\\ Triacylglycerols contain: \\\ amino acids \\ glycerol and fatty acid residues \\\ glucose \\\ fructose \\\\ If the BMI of adult person is 32,5, his weight is: \\\ ideal \\\ less then ideal \\\ excessive \\\ this is obesity \\\\ Which of the following is correct concerning the processes in the liver? \\\ Triacylglycerols are stored \\\ The process of glucose uptake is activated under the influence of glucagon \\ glucose is stored like glycogen \\\ glucose is converted into fatty acids \\\\ Vitamins in the body act as: \\\ transporters \\\ energy source \\\ enzymes \\ coenzymes \\\\ Which of the following processes occur during starvation? \\\ Glucose level doesnt change \\ activates liver glycogen degradation \\\ activates glycogen biosynthesis \\\ storage of fats increases \\\\ Glucose is only source of energy for: \\ erythrocytes \\\ nervous cells \\\ adipocytes \\\ muscle cells \\\\ Liver regulates glucose level in the body by: \\ glycogenolysis and glyconeogenesis \\\ oxidation of fatty acids \\\ oxidation of glycerol \\\ oxidation of ketone bodies

\\\\ For the functioning of the body we need the sufficient amount of the energetical sources. If we cant receive the sufficient energy with the food, the energy will be released from endogenic stores of the body, what provides: \\\ weight gaining \\ weight loss \\\ there is no change in weight \\\ neither answer is correct \\\\ Which of the following is monosaccharide? \\\ starch \\\ sacrose \\\ lactose \\ glucose \\\\ Body Mass Index (BMI) is calculated \\\ Kg2|m2 \\\ Kg2|mK \\ Kg|m2 \\\ Kg|m \\\\ Increased concentration of hydrogen ions provides: \\\ increase affinity of O2 to deoxiHb \\\ conversion of T form of Hb into R form of Hb in tissues \\ disociaiton of O2 from Hb \\\ conversion of HbOw ( not(deoxiHb) into Mb. \\\\ Which of the following contains the highest calories? \\ Lipids \\\ Proteins \\\ Carbohydrates \\\ Alcohol \\\\ Which is correct about starch? \\\ is disaccharide \\ is plant polysaccharide \\\ is animal polysaccharide \\\ is monosaccaride \\\\ Catabolic pathways are: \\ oxidation of energetical sources \\\ mobilisation of energetical sources \\\ biosynthetic processes \\\ detoxification \\\\ Which is the correct relation between the digestive enzymes and their substrates? \\\ pepsin triacylglicerol \\ amilase starch \\\ lipase sacrose \\\ tripsin lactose \\\\ The main energetical source for the body during overnight starvation is: \\\ Glycogen \\\ Glucose \\\ aminoacids \\ fatty acids \\\\ How many energy is released after oxidation of ethanol?

\\\ 4 kcal/g \\\ 5kcal/g \\\ 10 kcal/g \\ 7 kcal/g \\\\ The height is 170 sm and weight is 60 kg, what is BMI? \\\ 25,42 \\\ 20,76 \\\ 19,28 \\\ 23,64 \\\\ Which compounds are not essential? \\\ Fatty acids \\\ vitamins \\ glucose \\\ minerals \\\\ What are triacylglycerls? \\\ proteins \\ lipids \\\ carbohydrates \\\ vitamins \\\\ Why the protons dissociate from the hemoglobin? \\\ because pKa of the acid groups decrease \\\ because pKa of the acid groups increase \\\ because partial pressure of O2 decresases \\\ because concentration of CO increases. \\\\ How the fats are stored in the body? \\ as triacylglycerol \\\ fatty acids \\\ diacylglicerol \\\ cholesterol \\\\ which process is stimulated after carbohydrate rich diet? \\\ glucagon secretion \\\ glyconeogenesis \\\ oxidation of fatty acids \\ insulin secretion \\\\ Which cell forms ATP only with anaerboc pathway? \\ Erythrocutes \\\ Myocytes \\\ hepatocytes \\\ adipocytes \\\\ for calculation of BMI important parameters are: \\\ Body weight and age \\\ age, sex and physical activity \\\ height and physical activity \\ Body weight and height \\\\ Body weight loosing from the bigining is caused by: \\\ degradation of muscle proteins \\\ degradation of triqcylglycerols of adipose tissue \\ degradation of muscle and liver glycogen

\\\ decreaseing physical activity \\\\ Which is the correct relation between the digestive enzymes and their substrates? \\ pepsin protein \\\ amilase triacylglicerol \\\ lipase starch \\\ tripsin lactose \\\\ Which of the following does not belong to sources of energy? \\ vitamins \\\ proteins \\\ carbohydrates \\\ fats \\\\ Which of the following does not belong to sources of energy? \\\ proteins \\\ carbohydrates \\\ fats \\ minerals \\\\ Which of the following is the monomer of proteins? \\\ glucose \\\ glycerol \\ amino acid \\\ fatty acid \\\\ Which of the following is monosaccharide? \\\ starch \\\ sacrose \\\ lactose \\ glucose \\\\ Which of the following is monosaccharide? \\\ isomaltose \\\ sacrose \\\ lactose \\ fructose \\\\ Which of the following is monosaccharide? \\\ isomaltose \\\ sacrose \\\ lactose \\ galactose \\\\ Which compouns can serve as energy source for brain in fasting state? \\\ glycogen \\\ triacylglycerols \\\ proteins \\ ketone bodies \\\\ What statement is correct concerning anabolism? \\\ It is oxidation of energetical sources \\\ It is mobilization of energetical sources \\ It is biosynthetic pathway \\\ It is detoxication pathway \\\\ What statement is correct concerning catabolism?

\\ It is oxidation of energetical sources \\\ It is storage of energetical sources \\\ It is biosynthetic pathway \\\ It is detoxication pathway \\\\ How the organism store carbohydrates? \\ as glycogen \\\ as starch \\\ as sacrose \\\ as lactose \\\\ What is the substrate of amylase? \\\ isomaltose \\\ lactose \\ starch \\\ sacrose \\\\ Which of the following is not correct concerning glucose metabolism? \\\ it is oxidized in most of the cells \\\ it is stored in the liver and muscles as a glycogen \\\ it is the biosynthetic precursor of many other compounds \\ in the body it is produced from palmitate \\\\ Which is not correct concerning amino acids absorbed in the blood? \\\ in the cells they are converted into proteins \\\ they are used for the synthesis of nitrogen-containing compounds \\\ they are oxidized with reliese of energy \\ in the body they are stored as a glycogen \\\\ Which process is stimulated after high-carbohydrate containing meal? \\\ glucagon secretion \\\ gluconeogenesis \\\ oxidation of fats \\ insulin secretion \\\\ In which lipoprotein the liver packs triacylglycerols? \\ VLDL \\\ chylomicrons \\\ LDL \\\ HDL \\\\ In which organ the ketone bodies are synthesized? \\\ in muscles \\ in liver \\\ in kidneys \\\ in the brain \\\\ Which one of the following processes go in the liver? \\\ storage of triacylglyceros \\\ glycogen synthesis enhances under glucagon \\ glucose storage in form of glycogen \\\ ketone bodies oxidation \\\\ Which scheme is correct concerning digestive process? \\\ monosacharides-disacharides-polysacharides \\\ amino acids - di- and tripeptides - proteins \\ triacylglycerols- diacylglycerols - monoacylglycerols

\\\ chylomicrons - proteins - triacylglycerols \\\\ Which scheme is correct concerning digestive process? \\ polysacharides -disacharides- monosacharides \\\ amino acids - di- and tripeptides - proteins \\\ diacylglycerols- triacylglycerols - monoacylglycerols \\\ chylomicrons - proteins - triacylglycerols \\\\ Where does the resynthesis of triacylglycerols for chilomicrons go? \\\ in the adipose tissue \\\ in the blood \\ in the enterocytes' epithelium \\\ in the liver \\\\ How many energy is released after oxidation of 1g. of proteins? \\\ 2 kcal/g \\ 5kcal/g \\\ 9 kcal/g \\\ 7 kcal/g \\\\ How many energy is released after oxidation of 1g. of fats? \\\ 2 kcal/g \\\ 5kcal/g \\ 9 kcal/g \\\ 7 kcal/g \\\\ Which fatty acid is essential? \\\ palmitic acid \\ linoleic acid \\\ stearic acid \\\ oleic acid \\\\ Which fatty acid is essential? \\\ palmitic acid \\ arachidone acid \\\ stearic acid \\\ oleic acid

Buffers, water
\\\\ The pH of a sample of blood is 7.4, while gastric juice is pH 1.4. The blood sample has: \\\ 5.29 times lower [H+] than the gastric juice. \\\ 6 times lower [H+] than the gastric juice. \\\ 6,000 times lower [H+] than the gastric juice. \\ a million times lower [H+] than the gastric juice. \\\\ Which of the following describes a universal property of buffers? \\\ Buffers work best at the pH at which they are completely dissociated. \\ Buffers work best at the pH at which they are 50% dissociated. \\\ Buffers work best at one pH unit lower than the pKa. \\\ Buffers work equally well at all concentrations. \\\\ Which of the following physiologic/pathologic conditions is most likely to result in an alkalosis, provided

that the body could not fully compensate? \\\ Production of lactic acid by muscles during exercise \\ Repeated vomiting of stomach contents, including HCl \\\ Diarrhea with loss of the bicarbonate anions secreted in to the intestine \\\ An infection resulting in a fever and hypercatabolism
\\\\ \\\ \\\ \\ \\\

In a polypeptide at physiologic pH, hydrogen bonding may occur between: the side chains of a leucine residue and a lysine residue. the terminal alpha-amino group and the terminal a-carboxyl group. the amide group in the peptide bond and an aspartyl side chain. the SH groups of two cysteine residues.

\\\\ The concentration of hydrogen ions in a solution is measured by the pH, that numerically is equivalent to: \\\ log [H+]
10

\\ \\\ \\\

-log [H+] log [H+]


e 10

- log [H+]
e

\\\\ The pKa for ammonium ion is about 9.4 and the pH of blood is about 7.4. The ratio of ammonia to

ammonium ion (ammonia/ammonium ion) in blood is about: \\ 1/100 \\\ 1/10 \\\ 10/1 \\\ 100/1
\\\\ \\ \\\ \\\ \\\

Which of the following is least likely to be soluble in water? nonpolar compound strongly polar compound weak electrolyte strong electrolyte

\\\\ Which of the following is not characteristic for water? \\\ It is a polar molecule \\\ It is an excellent solvent \\\ It has a high surface tension \\ It interacts well with nonpolar molecules
\\\\ Hydrogen bonds can be expected to form with hydrogen atom bonded to:

\\\ carbon \\ electronegative atom \\\ hydrogen \\\ iodine

\\\\ Which of the following processes does not describe the action of NAD+? \\\ It is an oxidizing agent. \\ It is a reducing agent. \\\ It is an electron acceptor. \\\ It's reducing form is NADH+H+. \\\\ All of the following are considered "weak" interactions in proteins except: \\\ van der Waals forces. \\\ hydrogen bonds. \\ peptide bonds. \\\ hydrophobic interactions. \\\\ Water (H2O) has a bent shape, resulting in an unequal distribution of the electrons. Oxygen, which is more electronegative than the hydrogens, has a greater pull on the electrons, giving the oxygen a partial negative charge and the hydrogens a partial positive charge. This unequal distribution is called a: \\\ Covalent bond \\\ Tail \\\ Bent geometry \\ Dipole \\\\ The pH of a solution is the negative logarithm of the concentration of H+ ions. With this in mind, which solution has the greatest concentration of H+ ions? \\ A solution with pH=1 \\\ A solution with pH=7 \\\ A solution with pH=2 \\\ A solution with pH=10 \\\\ A sequence of amino acids in a certain protein is found to be SerGlyProGly. The sequence is most probably part of a(n): \\ beta- turn. \\\ parallel beta sheet. \\\ alpha sheet. \\\ antiparallel beta sheet.
\\\\ Which of the following is not correct concerning the water:

\\\ Water is donor, as well as acceptor of hydrogen ions \\ Each molecule of water can form 5 hydrogen bonds \\\ high boiling temperature is characteristic for water \\\ Boiling temperature of water is 0 C \\\\ Henderson-Haselbach equation presents relation between: \\\ pH of the solution and Ka of the acid \\ pH of the solution, Ka of the acid and dissociations level \\\ pKa of the acid and Kh of hydratation \\\ pH of the solution and Kh of hydratation \\\\ Blood pH is: \\\ 1,5-2,5 \\\ 5,5-6,0 \\ 7.36-7.44 \\\ 8.36-9.15

\\\\ Which is correct about acidic environment? \\ Concentration of H+ is more than concentration of OH\\\ Concentration of H+ is less than concentration of OH\\\ Concentration of H+ and OH- are equal \\\ pH = 8 \\\\ Which of the following is not correct about blood pH: \\\ in case of frequent breath blood pH increases \\ increased concentration of CO2 rises blood pH \\\ increased concentration of CO2 increases breath frequency \\\ increased concentration of CO2 decreases blood pH
\\\\ Buffers consist of:

\\\ strong acid and strong base \\ weak acid and conjugated base \\\ weak acid and weak base \\\ weak acids \\\\ Protein kinase phosphorylates hydroxy-groups. Which amino acids contain hydroxy group? \\\ Aspartate, glutamate, serine \\ Serine, treonine, tyrosine \\\ treonine, phenylalanine, arginine \\\ lysine, arginine, proline \\\\ Which is not correct about dissociation capacity of weak acids? \\\ pKa is negative logarithm of Ka \\\ as high is Ka, as strong is organic acid \\\ as low is pKa, as weak is acid \\ as high is pKa, as weak is acid. \\\\ Carboanhydrase catalyses: \\\ dissociation of carbonic acid into H+ and HCO3\\ degradation of carbonic acid into CO2 and H2O \\\ Protonation of Hb \\\ formation of carbaminohemoglobin \\\\ Which buffer system is not occure in the blood? \\\ Hemoglobin \\\ bicarbonate \\\ protein \\ amonium \\\\ Which of the following are the main electrolytes of the extracellular matrix? \\\ K+ and SO42\\ Na+ and Cl\\\ Mg2+ and HPO42-; \\\ Ca2+ and F2\\\\ pH is: \\\ log [H+] \\\ ln [H+] \\ - log [H+] \\\ - log[OH-] \\\\ Which of the following is not correct concerning the distribution of the water in the body? \\\ distribution of the water in the compartments is regulated by the osmolarity \\\ Water is moved from the low concentration to high concentration.

\\ Water is moved from the high concentration to low concentration. \\\ Hyperglycemia increase the osmotic pressure of the blood and this is cause of wter movement from the tissues into the blood \\\\ Which of the following is not correct concerning carboanhydrase? \\\ it catalyses formation of carbonic acid by connecting H+ and HCO3-. \\\ It fulfils the important role in the process of elimination of CO2. \\ It converts the main part of CO2 to carbonic acid. \\\ it catalyses dehydratation of carbonic acid and CO2 is produced. \\\\ What is buffer? \\ complex of nondissociated acid and conjugated base \\\ complex of dissociated acid and conjugated base \\\ complex of nondissociated acid and nonconjugated base \\\ complex of dissociated acid and nonconjugated base \\\\ Which is wrong about acids? \\ Strong acids dont dissociate completely \\\ Strong acids dissociate completely and give rise to H+ and anion compound \\\ Weak acids dissociate partly (noncompletely) \\\ As more is proton dissociation capacity, as high is its Ka \\\\ What is characteristic for water? \\\ linear structure \\ nonlinear bended structure \\\ helix structure \\\ hexsaedral structure \\\\ Which of the following is not correct about buffers? \\ As low is buffer concentration, as high is its buffer capacity \\\ they contain weak acids and conjugated base \\\ each buffer has its pH \\\ buffer system is effective, when the systems pH=pKa \\\\ What is the pH of water? \\\ 8 \\\ 9 \\\ 10 \\ 7 \\\\ Which of the following is characteristic for water, as for the best solvent? \\\ High boiling temperature \\\ low boiling temperature \\ hydratation capacity \\\ high evaporation capacity
\\\\ What is the pH of gastric juice?

\\ 1,5 - 2,5 \\\ 5,5 6,0 \\\ 7,36 7,44 \\\ 8,36 9,15
\\\\ Which are the main electrolytes of intracellular fluid?

\\ K+ and HPO42-; \\\ Na+ and Cl-; \\\ Mg2+ and SO42-; \\\ Ca2+ and F2-;

\\\\ Water content in the body is:

\\\ 40% \\ 60% \\\ 80% \\\ 100%


\\\\ Extracellular fluid is contributed in the following compartments: \\\ Only in the lymph \\\ Only in the blood \\\ Only in the interstitial fluid \\ In the blood, lymph and interstitial fluid

\\\\ Which is correct about extracellular fluid? \\\ 40 % of body's whole water is extracellular fluid \\\ serum belongs to intracellular fluid \\\ urine belongs to interstitial fluid \\\ 60 % of body's whole water is extracellular fluid \\\\ How many hydrogen bonds can be formed by each molecule of water? \\\ 2 \\\ 3 \\ 4 \\\ 5 \\\\ according to the acid-base theory, what is the acid? \\ H+ donor \\\ H+ acceptor \\\ OH- donor \\\ OH- acceptor

Signal transduction
\\\\ The effects of acetylcholine on the postsynaptic ion channel are mainly due to: \\\ protein cleavage (proteolysis). \\ protein conformational changes. \\\ protein phosphorylation. \\\ protein synthesis. \\\\ Protein kinase A (PKA) is: \\\ activated by covalent binding of cyclic AMP. \\ allosterically activated by cyclic AMP. \\\ competitively inhibited by cyclic AMP. \\\ noncompetitively inhibited by cyclic AMP. \\\\ Which of the following is not involved in signal transduction by the beta-adrenergic receptor pathway? \\\ ATP \\\ Cyclic AMP \\ Cyclic GMP \\\ GTP \\\\ Which of the following does not involve cyclic AMP? \\\ Regulation of glycogen synthesis and breakdown \\\ Regulation of glycolysis \\ Signaling by acetylcholine \\\ Signaling by glucagon \\\\ Hormone-activated phospholipase C can convert phosphatidylinositol 4,5-bisphosphate to: \\ diacylglycerol + inositol triphosphate. \\\ glycerol + inositol + phosphate. \\\ glycerol + phosphoserine. \\\ phosphatidyl glycerol + inositol + phosphate. \\\\ What is Calmodulin? \\\ allosteric activator of calcium-dependent enzymes. \\\ allosteric inhibitor of calcium-dependent enzymes. \\\ cell surface calcium receptor. \\ regulatory subunit of calcium-dependent enzymes.

\\\\ Which of the following is a general characteristic of all chemical messengers? \\\ They are secreted by one cell, enter the blood, and act on a distant target cell. \\\ To achieve a coordinated response, each messenger is secreted by several types of cells. \\ Each messenger binds to a specific protein receptor in a target cell. \\\ Chemical messengers must enter cells to transmit their message. \\\\ Which of the following is a characteristic of chemical messengers that bind to intracellular transcription factor receptors? \\\ They are usually cytokines or polypeptide hormones. \\\ They exert rapid actions in cells. \\ They are transported through the blood bound to proteins. \\\ They are always present in high concentrations in the blood. \\\\ SH2 domains on proteins are specific for which of the following sites? \\ Certain sequences of amino acids containing a phosphotyrosine residue \\\ PI-3,4,5 trisphosphate in the membrane \\\ Ca2+-calmodulin \\\ Receptor domains containing phosphoserine residues \\\\ Which of the following is incorrect for acetylcholine receptor? \\\ It is gatedion channel receptor \\ It is gene-specific transcriptional factor

\\\ binding of acetylcholine to the receptor opens the closed conformation. \\\ irritation of the receptor in the nerve-muscular synapsis results in muscle contraction \\\\ Which of the following is the first messenger? \\\ cAMP \\\ DAG \\ epinephrine \\\ IP3 \\\\ Which of the following is the second messenger? \\ cAMP \\\ insulin \\\ epinephrine \\\ glucagon \\\\ Which of the following is cytoplasmic receptor? \\\ gated ion channel receptor \\ gene-specific transcriptional factor \\\ Heptahelical receptor \\\ Tyrosine-kinase receptor \\\\ Which of the following statements is not characteristic for protein phosphatases? \\\ Protein phosphatases remove phosphate groups from proteins that have been phosphorylated \\\ Insulin activates protein phosphatases and removes phosphate groups added by the cAMP cascade \\\ The cAMP cascade can inhibit phosphatases \\ It takes a long time for the phosphatases to reverse the action of the cAMP cascade \\\\ G.M. has been fasting (500 kcal/day) for several days. Which one from the following statements would be decreased? \\\ Cyclic-AMP in adipose tissue \\\ The activity of hormone sensitive lipase in adipose tissue \\ The concentration of glycerol phosphate in adipose tissue \\\ The amount of free fatty acid entering the liver mitochondria \\\\ Which of the following is the first messenger? \\\ GMP \\\ DAG \\ glucagon \\\ IP3 \\\\ Which of the following is the second messenger? \\ DAG \\\ cortisole \\\ insulin \\\ epinephrine \\\\ Which of the following does not involve cyclic AMP? \\\ regulation of glycogen synthesis and breakdown \\\ regulation of glycolysis \\\ signaling by epinephrine \\ signaling by acetylcholine \\\\ In which pathway of signal transduction the signal molecule is carried by the blood? \\ Endocrine \\\ Paracrine \\\ Sinaptic \\\ Autocrine

\\\\ Which signal moleculs target cells receptor has the high affinity to the ligand? \\\ Local mediators \\\ Neurotransmitters \\\ Hormones \\\ Large polipeptide growth factors
\\\\ Which of the following molecule cant cross the cell plasma membrane? \\\ Steroid hormone \\ Growth factor \\\ Thyroid hormone \\\ Retinoic acid \\\\ Which of the following is lipid second messenger? \\\ cAMP \\ diacylglycerol (DAG) \\\ cGMP \\\ Ca2+ \\\\ Which enzymes catalyze proteins phosphorylation in the intracellylar signal transduction? \\\ Phosphodiestherases \\\ Protein phosphatases \\ Protein kinases \\\ Phosphohydrolases \\\\ What is desensitization? \\ Loss of sentitivity of target cells receptors towards agonists \\\ The starting step of signal transduction \\\ Receptors increased affinity towards agonist \\\ Signal propogation inside the cell \\\\ Which description is correct concerning G protein cycle? \\\ In basal state G proteins are homotrimeres \\ Alpha subunit has GTP-ase activity \\\ Alpha- GTP is nonactive conformation towards effector \\\ Alpha- GDP is active conformation towards effector \\\\ Which of the following is the major messenger in the relaxation process of blood vessels? \\\ DAG \\\ PIP3 \\\ cAMP \\ cGMP \\\\ What is called the type of intercellular signaling, when it occures between the distant cell? \\\ Autocrine \\ Endocrine \\\ Contact-dependent \\\ Paracrine \\\\ Which statement is correct concerning intracellular receptors? \\\ As a rule they connect to hydrophilic ligands \\\ They are situated in the cytosol in complex with protein \\ After connecting to ligand, they regulate gene transcription \\\ Have ability to connect DNA specific consequences without connection to ligand \\\\ What of the following is correct concerning membrane receptors? \\\ It connects only with large molecules \\ After binding to ligand it activates enzyme cascade \\\ After binding to protein opens the ionic chanells \\\ Produces second messengers without binding with proteins \\\\ Which statement opposites of the signal transduction pathway based on the protein phosphorylation?

\\ Tyrosin kinase is always part of receptor protein \\\ Phosphorylation may go on Serine or Threonine \\\ protein kinase cascade causes signal amplification \\\ phosphorylation changes protein structure and function \\\\ How the termination of signal transduction go with help of cell membrane receptors? \\\ With increasing agonist concentration \\ With internalization and degradation of receptor-agonist complex \\\ With increasing the receptor sensitivity towards agonist \\\ With increasing the amount receptors \\\\ What is Calmodulin? \\\ Nonspecific kinase \\ Ca2+-binding protein \\\ Second messenger \\\ Protein channel which is activator of Ca+ influx \\\\ Which statement is correct about G protein? \\\ G protein is directly bound to the hormone on the cell surface \\ Alpha subunits may have stimulatory activity, as well as inhibiting activity \\\ Hydrolysis of GTP is essential for separation of GTP \\\ in the resting state GTP is bound with G-protein \\\\ Which amino acids radicals are the subjects of phosphorylaton during protein kinase cascade? \\\ Glycine and valine \\\ Proline and methionine \\\ Tripthopan and phenylalanine \\ Serine and threonine \\\\ Which amino acid radical is the subject of phosphorylaton during protein kinase cascade? \\\ valine \\\ phenylalanine \\\ Tripthopan \\ tyrosin \\\\ What is not thought in the termination mechanism of signal transduction? \\\ decreasing the agonist concentration in the environment \\\ Internalization of agonist-receptor complex \\\ Receptor inactivation \\ increasing affinity between agonist and receptor \\\\ What is not characteristic for tyrosin kinase receptors? \\ Existence of several transmembrane domeins \\\ Dimerization after binding with ligand \\\ Autophosphorylation \\\ binding to intracellular specific proteins with recognizing site \\\\ High affinity of target cells receptors towards hormones provides: \\\ Formation of contact dependent transduction of signal \\\ binding of any hormone despite of their concentration in the blood \\ Specific binding of any hormone even in case of their low concentration in the blood \\\ Easy dissociation of hormone from hormone-receptor complex \\\\ Which is not second messenger? \\ Epinephrine \\\ cAMP \\\ diacylglycerol \\\ PIP3 \\\\ Which signal molecules are used for signal transduction in synaptic way? \\\ Citokines \\ Neurotransmitters \\\ Steroid hormones

\\\ Prostaglandines \\\\ From the following which one has no intracellular receptors? \\\ Derivatives of Vit.D3 \\ Epinephrine \\\ Thyroid hormones \\\ Steroid hormones \\\\ Which statement is correct concerning second messengers? \\ They are small sized molecules, which are generating after binding agonist with receptor and provide signal intracellular transduction \\\ They are large sized molecules, which transmit signals to target cells \\\ All of them are water soluble \\\ All of them are organic molecules \\\\ Which signal molecule is responsible for inflamatory and immune response? \\\ Neurotransmitter \\\ Protein hormones \\ Cytokines \\\ Steroid hormones \\\\ Which statement is not true concerning heterotrimeric G protein? \\\ It is coupled with 7 transmembrane containing receptor \\\ Contains with alpha, beta, gama subunits \\\ Alpha subunit has guanin nucleotide-binding and GTP-ase acitvity \\ s subunits of Gs proteins inhibit adenylatcyclase \\\\ Which enzyme increases intracellular concentration of cAMP? \\\ Phosphoprotein phosphatases \\ Adenylate cyclase \\\ Guanylate cyclase \\\ Cyclic nucleotide phosphodiesterase \\\\ To which enzymes activity is connected NO? \\ Soluble guanylate cyclase \\\ cGMP-dependent proteinkinase \\\ cAMP-dependent proteinkinase \\\ adenylate cyclase \\\\ Which neurotransmitter regulates anion-selective receptors? \\ Acetylcholine \\\ Gama- aminobutirate \\\ Glutamate \\\ serotonine \\\\ Which does not belong to plasma membrane receptors? \\\ Ion-channel type receptors \\ Gene-specific transcriptional factor \\\ Heptahelical receptors \\\ Tyrosin kinase receptors \\\\ Which receptor does not respond to epinephrine with activation of cAMP cascade? \\\ Beta1-adrenergic receptor \\\ Beta2-adrenergic receptor \\ Alpha1-adrenergic receptor \\\ Beta3-adrenergic receptor \\\\ Which is the second messenger? \\ cAMP \\\ Cortisole \\\ Insulin \\\ Epinephrin

\\\\ Functioning of PIP system starts with phospholipase C activation, what initiates the proceeding of concequent events. Which of the following is not relevant for this process? \\ IP3 releases with help of phopholipase A \\\ Increase concentration of intracellular concentration of Ca2+ \\\ Activation of proteinkinase A \\\ phophorylation of cytoplasmic proteins

Proteins, amino acids


\\\\ Which is characteristic concerning chymotripsin? \\ Chymotripsin is a serine-protease \\\ Chymotripsin hydrolyzes peptide bonds containing alanine, glycine and proline \\\ The covalent intermediates are formed before the formation of enzyme-substrate complex. \\\ Lysine is providing the reactive group for the active site.
\\\\ Which amino acid from 20 standard amino acids is optically inactive? \\\ alanine \\ glycine; \\\ valine \\\ lysine \\\\ What is the reason of optical inactivity of glycine? \\\ Side chain contains a simple methyl group \\ Side chain contains a hydrogen atom \\\ Side chain contains unbranched \\\ Side chain forms a covalent bond with the amino group \\\\ Which two amino acids of the standard 20 contain sulfur atoms? \\\ cysteine and serine. \\\ cysteine and threonine. \\ methionine and cysteine \\\ methionine and serine \\\\ What group is characteristic for all of the amino acids that are found in proteins, except for proline? \\ amino group. \\\ carbonyl group. \\\ carboxyl group. \\\ ester group. \\\\ Which of the following statements about cystine is correct? \\ Cystine forms when the CH2SH R group is oxidized to form a CH2SSCH2 disulfide bridge between two cysteines. \\\ Cystine is an example of a nonstandard amino acid, derived by linking two standard amino acids. \\\ Cystine is formed by the oxidation of the carboxylic acid group on cysteine. \\\ Two cystines are released when a CH2SSCH2 disulfide bridge is reduced to CH2SH. \\\\ Why amino acids are called as ampholytes? \\ they can function as either an acid or a base. \\\ they can function as either a neutral molecule or an ion. \\\ they can function as either a polar or a nonpolar molecule. \\\ they can function as either a transparent or a light-absorbing compound. \\\\ In an aqueous solution, protein conformation is determined by two major factors. One is the formation of the maximum number of hydrogen bonds. What is the other reason? \\\ formation of the maximum number of hydrophilic interactions. \\\ minimization of entropy by the formation of a water solvent shell around the protein. \\ placement of hydrophobic amino acid residues within the interior of the protein. \\\ placement of polar amino acid residues around the exterior of the protein. \\\\ What is the reason of sickle cell anemia? \\ substitution of Glu with Val in beta-chain of hemoglobin \\\ substitution of Met with Asp in beta-chain of hemoglobin \\\ substitution of Leu with ILe in beta-chain of hemoglobin \\\ substitution of Phe with Gln in beta-chain of hemoglobin \\\\ How in the alpha helix the hydrogen bonds are arranged? \\ are roughly parallel to the axis of the helix. \\\ occur mainly between electronegative atoms of the R groups. \\\ occur only between some of the amino acids of the helix.

\\\ occur only near the amino and carboxyl termini of the helix. \\\\ How in an alpha helix, the R groups on the amino acid residues are arranged? \\\ alternate between the outside and the inside of the helix. \\ are found on the outside of the helix spiral. \\\ cause only right-handed helices to form. \\\ generate the hydrogen bonds that form the helix. \\\\ A D-amino acid would interrupt an alpha helix made of L-amino acids. What is the naturally occurring hindrance to the formation of an helix? \\\ a negatively charged Arg residue. \\\ a positively charged Lys residue. \\ Pro residue. \\\ two Ala residues side by side. \\\\ Proteins often have regions that show specific, coherent patterns of folding or function. What these regions are called? \\ domains. \\\ oligomers. \\\ sites. \\\ subunits. \\\\ Which of the following is known not involved in the process of folding of proteins? \\\ Chaperonins \\\ Disulfide interchange \\\ Heat shock proteins \\ Peptide bond hydrolysis \\\\ Which one describes an allosteric interaction between a ligand and a protein? \\\ binding of a molecule to a binding site affects binding of additional molecules to the same site. \\ binding of a molecule to a binding site affects binding properties of another site on the protein. \\\ binding of the ligand to the protein is covalent. \\\ two different ligands can bind to the same binding site. \\\\ Which of the following secondary structures is most likely to be found in a membrane-embedded portion of a protein? \\ An alpha helix composed entirely of hydrophobic residues \\\ An amphipathic alpha helix \\\ An open beta sheet composed of antiparallel beta strands \\\ An open beta sheet composed of parallel beta strands

\\\\ Which of the following shows the linear sequence of atoms joined by covalent bonds in a peptide backbone? \\ -N-C-C-N-C-C-N-C-C\\\ -N-C-O-N-C-O-N-C-O\\\ -N-H-C-C-N-H-C-C-N-H-C-C \\\ -N-H-C-O-H-N-H-C-O-H-N-C-C\\\\ Different preparations of insulin contain some insulin complexed with protamine that is absorbed slowly after injection. Protamine is a protein preparation from rainbow trout sperm containing arginine-rich peptides that bind insulin. Which of the following provides the best explanation for complex formation between protamine and insulin? \\ Arginine is a basic amino acid that binds to negatively charged amino acid side chains in insulin. \\\ Arginine is a basic amino acid that binds to the alpha-carboxylic acid groups at the N-terminals of insulin chains. \\\ Arginine is a large bulky hydrophobic amino acid that complexes with leucine and phenylalanine in insulin. \\\ Arginine forms disulfide bonds with the cysteine residues that hold the A and B chains together. \\\\ Protein kinases phosphorylate proteins only at certain hydroxyl groups on amino acid side chains. Which of the following groups of amino acids all contain side chain hydroxyl groups?

\\\ \\ \\\ \\\

aspartate, glutamate,and serine serine, threonine, and tyrosine threonine, phenylalanine, and arginine lysine, arginine, and proline

\\\\ Which of the following is a characteristic of globular proteins? \\\ Hydrophilic amino acids tend to be on the inside. \\\ Hydrophobic amino acids tend to be on the outside. \\ Tertiary structure is formed by hydrophobic and electrostatic interactions between amino acids, and by hydrogen bonds between amino acids and between amino acids and water. \\\ secondary structures are formed principally by hydrophobic interactions between amino acids. \\\\ Autopsies of patients with Alzheimer's disease show protein aggregates called neurofibrillary tangles and neuritic plaques in various regions of the brain. These plaques exhibit the characteristic staining of amyloid. Which of the following structural features is the most likely characteristic of at least one protein in these plaques? \\ A high content of beta-pleated sheet structure \\\ A high content of alpha-helical structure \\\ A high content of random coils \\\ Disulfide bond crosslinks between polypepride chains \\\\ Which of the following amino acid interrupts helical conformation? \\\ Valine \\\ Alanine \\ Proline \\\ Cysteine \\\\ Which is incorrect for alphahelix? \\\ The side chains are on the outside of the spiral structure. \\ The side chains are directed toward the center of the helix. \\\ alphahelix is the right-handed structure with hydrogen bonds between the first and every fourth amino acid. \\\ consists of only one polypeptide chain. \\\\ Which of the following is incorrect for betastructure? \\\ The structure has a pleated-sheet-like form and the chains may have parallel, as well as antiparallel direction; \\\ There are inter-chain hydrogen bonds in the structure; \\ Interchain disulfide bonds are necessary for stabilization of the structure; \\\ increasing content of beta-structure causes increse in hydrophobisity of proteins. It explains PrPcs agregates formation during the encephalopathy induced by prion-proteins . \\\\ Which of the following best describes an alpha-helical region of a polypeptide? \\\ Left-handed, 5,4 aa/turn; \\\ Left-handed, 6,5 aa/turn \\ Right-handed, 3,6 aa/turn \\\ Right-handed, 7,0 aa/turn \\\\ Sulphur-containing amino acid is: \\\ Valine \\\ Phenylalanine \\ Methionine \\\ Leucine \\\\ Which of the following amino acids is synthesized only after incorporation of a precursor into a polypeptide? \\\ Lysine

\\ Hydroxyproline \\\ Glutamate \\\ Serine \\\\ Which of the following is the catalytic triad of chymotripsin? \\\ Phenylalanine-Lysine-Glycine \\\ Tyrosine-Isoleucine-Leucine \\ Aspartate-Histidine-Serine \\\ Arginine-Cysteine-Methionine \\\\ Glutathione is continuously being reduced and oxidized. Which group on the molecule is actually being reduced and oxidized: \\\ The hydroxyl group of threonine \\ The sulfhydryl group of cysteine \\\ The carboxyl group of aspartate \\\ The reactive hydrogen of histidine \\\\ Which of the following has quaternary structure? \\\ alpha-chymotripsin \\ hemoglobin \\\ myoglobin \\\ insulin \\\\ What is characteristic for chaperones? \\\ All require ATP to exert their effect. \\\ cleave incorrect disulfide bonds, allowing correct ones to subsequently form \\ are involved in the transport of proteins across mitochondrial and endoplasmic reticulum membranes \\\ act primarily on fully synthesized polypeptide chains, unfolding incorrect structures so they can refold correctly. \\\\ alphahelix and beta-sheet are the types of which below listed structures? \\\ primary \\ secondary \\\ tertiary \\\ quaternary \\\\ Which interaction is characteristic for primary structure? \\\ hydrogen bonding \\\ hydrophobic interaction \\\ van der Waals forces \\ peptide bonding \\\\ Which is positively charged amino acid? \\\ alanine \\ Arginine \\\ valine \\\ glutamate \\\\ Which is negatively charged amino acid? \\ aspartate \\\ lysine \\\ serine \\\ tyrosine \\\\ Which is positively charged amino acid? \\\ phenylalanine \\ histidine

\\\ valine \\\ tyrosine \\\\ Which is positively charged amino acid? \\ lysine \\\ aspartate \\\ glutamic acid \\\ proline \\\\ Which is neutral charged amino acid? \\\ histidine \\ leucine \\\ glytamate \\\ cystein \\\\ Which is neutral charged amino acid? \\\ lysine \\ isoleucine \\\ glutamate \\\ threonine \\\\ Which is neutral charged amino acid? \\\ histidine \\ valine \\\ aspartate \\\ arginine \\\\ Which amino acid contains OH group? \\\ valine \\\ arginine \\ tryptophan \\\ phenylalanine \\\\ Which amino acid contains OH group? \\\ isoleucine \\\ arginine \\ tyrosin \\\ phenylalanine \\\\ Which amino acid contains OH group? \\\ valine \\\ aspartate \\ serine \\\ glutamin \\\\ Which amino acid is aromatic? \\\ alanine \\ phenylalanine \\\ proline \\\ aspartate \\\\ Which of the following is imino acid? \\\ glicine \\\ isoleucine \\ proline \\\ asparagine

\\\\ Refer to the following structure for the following question: Gly Ser Cys Glu Asp Cys Arg | | S S

The peptide shown above: \\\ has arginine in position 1 of the sequence; \\ contains a derived amino acid; \\\ is basic \\\ consists entirely of amino acids with charged or nonpolar side chanes. \\\ contains more amino acids with side chains that are charged then ones with electrically neutral side chains at pH 7,0.

Fig. titraional curve \\\\ The figure above shows the titration curve of one of the common amino acids. From this curve we can conclude: \\\ the amino acid contains two carboxyl groups; \\ at point B amino acid is zwitterionic; \\\ the amino acid contains an aromatic hydroxyl group; \\\ point D corresponds to the pKa of an ionizable group; \\\ at point E the amino acis has a net negative charge. \\\\ NH3+- CH CO NH CH CO NH- CH-COO | | | CH2 CH2 CH-CH3 | | | COOSH CH2 | CH3
-

The peptide consists of the following amino acids: \\ Asp-Cys- Ile \\\ Met-Cys-Val \\\ Phe-Gly-Glu \\\ Asp-Met-Leu \\\\ The peptide consists of the following amino acids:

NH -CH CO NH CH CO NH- CH-CO NH CH2- COO3

| CH2OH

| CH2 |

| CH2 |

| OH \\ \\\ \\\ \\\ Ser-Phe-Tyr-Gly Thr-Tyr-Phe-ala Val- His- Arg- Cys Ser-Leu-Ile-gly

\\\\ 42-amino acids peptide related to the extracellular Alzheimer amyloid deposits has the last few residues immersed in the membrane bilayer. Based on your knowledge about membrane proteins, which of the following sequences most probably identifies the last five amino acids in this 42 residue peptide? \\\ Ala-Glu-Phe-Arg \\ Val-Val-Ile-Ala \\\ Asp-Ser-Gly-Tyr \\\ Lys-Val-His-His-Gln \\\\ Under physiological conditions, which of the following processes is not an important method for regulating the activity of enzymes? \\\ Phosphorylation. \\ Temperature changes. \\\ Adenyl addition. \\\ Disulfide reduction. \\\\ protein has one transmembrane domain, which is alpha-helix. which amino acid is feasible in this transmembrane domain? \\\ arginine \\\ glutamate \\\ lysine \\ leucine \\\\ In the process of proteins folding the critical role has: \\\ net charge of protein \\ noncovalent strength \\\ molecular weight \\\ polypeptide lenght \\\\ The net charge of Aspartate on pH 7,4 is: \\ -1 \\\ -2 \\\ 0 \\\ +1 \\\\ Which is characteristic for alphaspiral regions? \\\ Each has the same primary structure \\ alphaspiral forms with hydrogen bonds formed between the peptide bonds oxygen atom of carbonyl group and peptide bonds hydrogen atom of amide group \\\ they form with hydrogen bonds between the peptide bonds carbonyl atom and hydrogen atom of side chain.

\\\ they form with hydrogen bonds between the neighbor amino acids in primary structure. \\\\ Which is the correct concerning ionized groups? \\ pKa of acid group is not depend on the environment \\\ pKa of basic amino acids are high. \\\ pKa of acidic amino acids are high. \\\ acidic amino acids in physiological conditions are positively charged. \\\\ Which amino acids deficiency involving in collagen may be caused by deficiency of vitamin C? \\ Hydrxyproline \\\ allysine \\\ Cystein \\\ Methionine \\\\ Which answer is correct concerning globular proteins? \\\ Hemoglobin has 2-binding site of oxygen \\\ mioglobin is fibrous protein \\ hemoglobin is tetramer with quaternary structure \\\ mioglobin is quaternary structure protein. \\\\ Which amino acids are characteristic for elastin? \\\ Leucine and isoleucin \\\ alanine and phenylalanine \\ Desmosin and Isodesmosin \\\ asparagin and asparagine acid \\\\ Which derived amino acid is unique for collagen? \\\ Cystein \\\ Desmosin \\\ Isodesmosin \\ hydroxyprolin \\\\ Which is correct for collagen? \\\ Collagen spiral n=2 \\ Glycine is every third position \\\ spiral is stabilizes with intrachain hydrogen bonds \\\ transverse covalent bonds are formed with alanine modification \\\\ What is the function of fibrous proteins? \\\ transport \\ structural \\\ catalytic \\\ defense \\\\ Which amino acid content achieves to 33% in collagen molecule? \\ glycine \\\ valine \\\ asparagines \\\ glytamine \\\\ How many polypeptide chains contains alpha-keratin? \\\ 1 \\\ 5 \\\ 7 \\ 3 \\\\ Which is incorrect concerning elastin?

\\\ There are derived amino acids in elastin, among them desmosin and isodesmosin have high importance \\\ Elastin provides elasticity of tissues \\ It has regular secondary structure \\\ Elastin is in a high amount in the skin, gut, tendoms, lungs, etc. \\\\ Tertiary structure of the protein mean: \\\ sequence of amino acids \\ conformation of protein \\\ association of subunits \\\ type of regular structure \\\\ If the amino acid is frequently occur in the same positions of similar proteins of different species, the amino acid is: \\\ varable amino acid \\\ positively charged amino aci \\\ negatively charged amino acid \\ invariant amino acid \\\\ Which of the following does not belong to fibrous proteins? \\\ Collagen \\\ Tropomyosin \\ Hemoglobin \\\ Elastin \\\\ Which amino acid occur in a high amount in collagen? \\\ methionine \\ Prolin \\\ Triptophan \\\ Valine \\\ Asparagine \\\\ Which amino acid is feasible on the surface of globular proteins? \\ Serine \\\ Leucine \\\ Alanine \\\ Isoleucine \\\\ What is characteristic for quaternary structure of proteins? \\\ sequence of amino acids \\\ structural motives \\ asossiation of subunits \\\ contains disulfide bridges \\\\ Which is the wrong statement about heme? \\\ contains protoporphirin IX ring \\ heme molecule exist only in hemoglobin \\\ is nonprotein part \\\ Fe2+ in the centre of the moleculen make 6 coordinative bonds \\\\ Repetitive tandem sequences are characteristic for the following structure of proteins: \\\ Primary \\ Secondary \\\ Tertiary \\\ Quaternary \\\\ Which feature is not characteristic for fibrous proteins? \\\ high amount of secondary structures

\\ good solvation in water \\\ long thread-like form \\\ structural function \\\\ Which feature is characteristic for collagen? \\\ contains high amount of arginine \\ molecule is alpha-helix \\\ contains high amount of proline and glycine \\\ it is lipoprotein \\\\ How many polipeptide chains contain collagen (tropocollagen) molecule? \\\ 1 \\\ 2 \\\ 4 \\ 3 \\\\ Which particular amino acid is characteristic for elastin? \\\ Histidine \\\ Asparagine \\ Desmosin \\\ Valine \\\\ Which is characteristic for fibrous proteins? \\ bad solvation in water \\\ spherial form \\\ dinamic function \\\ globular form

Enzymes
\\\\ What is characteristic for competitive inhibitor? \\ increases Km without changes of Vmax \\\ decreases Km without changes of Vmax \\\ increases Vmax without changes of Km \\\ decreases both Vmax and Km \\\\ Which of the following is the oxidative-reduction coenzyme? \\ NAD+ \\\ pyridoxalphosphatre \\\ coenzyme A \\\ biotin \\\\ Which one of the following is not among the six internationally accepted classes of enzymes?
\\\ Hydrolases \\\ Ligases \\ Polymerases \\\ Transferases \\\\ Why are enzymes potent catalysts? \\\ They are consumed in the reactions they catalyze. \\\ They are very specific and can prevent the conversion of products back to substrates. \\\ They increase the equilibrium constants for the reactions they catalyze. \\ They lower the activation energy for the reactions they catalyze. \\\\ Which of the following statements is true of enzymes as catalysts? \\\ Their catalytic activity is independent of pH. \\\ They can increase the equilibrium constant for a given reaction by a thousand fold or more. \\ They can increase the reaction rate for a given reaction by a thousand fold or more. \\\ To be effective, they must be present at the same concentration as their substrate. \\\ What feature differs enzymes from other catalysts? \\\ are not consumed in the reaction. \\ display specificity towards a single reactant. \\\ form an activated complex with the reactants. \\\ lower the activation energy of the reaction catalyzed. \\\\ To what fact refers the concept of induced fit ? \\\ enzyme specificity is induced by enzyme-substrate binding. \\\ enzyme-substrate binding induces movement along the reaction coordinate to the transition state. \\ substrate binding may induce a conformational change in the enzyme, which then brings catalytic groups into proper orientation. \\\ when a substrate binds to an enzyme, the enzyme induces a loss of water (desolvation) from the substrate. \\\\ How does competitive inhibitor act? \\\ binds at several different sites on an enzyme. \\\ binds only to the ES complex. \\ binds reversibly at the active site. \\\ lowers the characteristic Vmax of the enzyme. \\\\ What is characteristic for allosteric enzymes? \\\ are regulated primarily by covalent modification. \\\ usually catalyze several different reactions within a metabolic pathway. \\ usually have more than one polypeptide chain. \\\ usually have only one active site.

\\\\ What type(s) of inhibition can be reversed? \\\ competitive \\\ noncompetitive

\\\ mixed \\ All of the above \\\\ Which of the following describes a characteristic of most allosteric enzymes? \\\ They are composed of single subunits. \\\ In the absence of effectors, they generally follow Michaelis-Menten kinetics. \\ They show cooperativity in substrate binding. \\\ They have allosteric activators that bind in the catalytic site. \\\\ What is Km ? \\\ equal to the product concentration at initial reaction conditions. \\ equal to the substrate concentration when the reaction rate is half its maximal value. \\\ proportional to the standard free energy. \\\ All of the above.
\\\\ Which of the following statements about buffers is true? \\\ A buffer composed of a weak acid of pKa = 5 is stronger at pH 4 than at pH 6. \\\ The pH of a buffered solution remains constant no matter how much acid or base is added to the solution. \\\ The strongest buffers are those composed of strong acids and strong bases. \\ When pH = pKa, the weak acid and salt concentrations in a buffer are equal. \\\\ Which of the following is characteristic for a competitive inhibitor of an enzyme-catalyzed reaction?

\\\ The inhibitor binds covalently to the enzyme. \\\ The inhibitor binds noncovalently to the substrate. \\ In the presence of the inhibitor, the reaction exhibits an apparent increase in KM. \\\ In the presence of the inhibitor, the reaction exhibits an apparent decrease in Vmax. \\\\ Which of the following best describes the type of reaction catalyzed by a kinase? \\\ Adenylylation \\\ Hydrolysis \\\ Phosphorolysis \\ Phosphorylation \\\\ Which statement from the following is true for the primary structures of enzymes catalyzing the same reaction in a single individual? \\\ are exactly the same from cell type to cell type, although the amount of enzyme may differ. \\\ stay the same throughout the lifetime of that individual. \\\ are identical if the enzymes are paralogs. \\ may differ between different cellular compartments of the same cell. \\\\ While studying a novel pathway in a remote species of bacteria, you discover a new globular protein that phosphorylates a substrate, using ATP as the phosphate donor. This protein most likely contains which of the following structures? \\ An actin fold \\\ An immunoglobulin fold \\\ A nucleotide binding fold \\\ A globin fold \\\\ Which of the following describes a characteristic feature of an enzyme obeying Michaelis-Menten kinetics? \\\ The enzyme velocity is at 1/2 the maximal rate when 100% of the enzyme molecules contain bound substrate. \\ The enzyme velocity is at 1/2 the maximal rate when 50% of the enzyme molecules contain bound substrate. \\\ The enzyme velocity is at its maximal rate when 50% of the enzyme molecules contain bound substrate. \\\ The velocity of the reaction is independent of the concentration of enzyme. \\\\ Methanol (CH3OH) is converted by alcohol dehydrogenases to formaldehyde (CH2O), a compound that is

highly toxic in the human. Patients who have ingested toxic levels of methanol are sometimes treated with ethanol (CH3CH2OH) to inhibit methanol oxidation by alcohol dehydrogenase. Which of the following statements provides the best rationale for this treatment? \\\ Ethanol is a structural analog of methanol, and might therefore be an effective noncompetitive inhibitor. \\ Ethanol is a structural analog of methanol that would be expected to compete with methanol for its binding site on the enzyme. \\\ Ethanol would be expected to alter the Vmax of alcohol dehydrogenase for the oxidation of methanol to formaldehyde. \\\ Ethanol would be an effective inhibitor of methanol oxidation regardless of the concentration of methanol. \\\\ Which of the following would NOT require NADPH? \\\ The reduction of antioxidant vitamins \\\ The reductive biosynthesis of cholesterol \\\ The biosynthesis of palmitate \\ The beta-oxidation of fatty acids \\\\ What is characteristic for active site of enzymes? \\\ contains the substrate-binding site \\\ is contiguous with the substrate-binding site in the primary sequence. \\\ contains a metal ion as a prostethic group \\ contains the amino acid side chains involved in catalyzing reaction. \\\\ Which of the following cofactors contains thiamin group? \\\ NAD \\\ FAD \\ TPP \\\ Pyridoxal phosphate \\\\ Drugs, that act as enzyme inhibitors: \\ may function as competitive inhibitors \\\ Unlike antibiotics, are free of the danger of drug resistance. \\\ Must be harmless to the patient \\\ Generally mimic the three dimensional structure of the enzymes active site. \\\\ Which of the following necessarily results in formation of an enzyme-substrate intermediate? \\\ substrate strain \\\ acid-base catalysis \\\ allosteric regulation \\ covalent catalysis \\\\ In which conditions enzymatic reaction becomes irreversible? \\ if the products are thermodynamically far more stable than the reactants. \\\ under equilibrium conditions \\\ at high enzyme concentrations \\\ at high temperature. \\\\ An holoenzyme is \\\ a coenzyme \\ an enzyme with its cofactor \\\ an enzyme lacking its cofactor \\\ a cofactor \\\\ Which of the following statements is true concerning enzymes, as catalysts? \\\ To be effective, they must be present at the same concentration as their substrate. \\\ They can increase the equilibrium constant for a given reaction by a thousand-fold or more. \\ They lower the activation energy for conversion of substrate to product.

\\\ Their catalytic activity is independent of pH. \\\\ The role of an enzyme in an enzyme-catalyzed reaction is to: \\\ ensure that the product is more stable than the substrate. \\\ make the free-energy change for the reaction more favorable. \\ increase the rate at which substrate is converted into product. \\\ ensure that all the substrate is converted to product. \\\\ A small molecule that decreases the activity of an enzyme by binding to a site other than the catalytic site is termed a(n): \\\ alternative inhibitor. \\ allosteric inhibitor. \\\ stereospecific agent. \\\ competitive inhibitor. \\\\ Enzymes are classified by the: \\\ Size of the enzyme \\\ Size of the substrate \\ Type of reaction they catalyze \\\ Rate of reaction \\\\ What is true regarding competitivive inhibition? \\\ Competitive inhibitor is is structural analog of substrate. \\\ Competitive inhibition is irreversible \\\ Increased substrate concentration doesnt reflect on this type of inhibition \\ Increased substrate concentration reverses this type of inhibition. \\\\ To which class of enzymes dehydrogenases belong? \\\ Transferases \\ Oxidoreductases \\\ lyases \\\ isomerases \\\\ Isozymes are not different with: \\\ Primary structure \\\ Physical-chemical features \\ Substrate specificity \\\ Tissue localization \\\\ Which class of enzymes requires energy for catalysis which releases from ATP or NTP hydrolysis? \\\ Transferases \\\ Oxidoreductases \\ ligases \\\ isomerases \\\\ Which coenzyme is involved in trasferase reactions? \\\ NAD+ \\\ Ubiquinone \\ Pyridoxalphosphate \\\ FAD \\\\ Which of the following statements is not true about enzymes, as catalysts? \\\ They increase the rate of chemical reaction million and milliard times \\\ They have high specificity \\ As a rule, they act during high temperature (100 degree of Celciuse) \\\ The rate of enzymatic reaction is directly proportional with enzyme concentration

\\\\ To which class of enzymes oxidases belong? \\\ Transferases \\ Oxidoreductases \\\ lyases \\\ isomerases \\\\ To which class of enzymes does lactatedehydrogenase belong? \\\ Transferases \\ Oxidoreductases \\\ lyases \\\ isomerases \\\\ Which class of enzymes catalyze biosynthetic reactions? \\\ Oxidoreductases \\\ Transferases \\ Synthetases \\\ Lyases \\\\ Which class of enzymes catalyze cleavage of intramolecular bindings with help of water? \\ Hydrolases \\\ Transferases \\\ Synthetases \\\ Lyases \\\\ Which class of enzymes catalyze transfer of any group between compounds? \\\ Lyases \\\ Ligases \\\ isomerases \\ Transferases \\\\ Which class of enzymes catalyze oxidative-reductive reactions? \\\ Transferases \\ Oxidoreductases \\\ lyases \\\ hydrolases \\\\ What feature is characteristic for active site of enzyme? \\ It has three-dimentional structure \\\ Maintains conformation during enzyme-substrate complex formation \\\ Only one amino acid is involved in the formation of active site \\\ Active site forms hydrophilic pocket in the enzyme molecule \\\\ What is characteristic for allosteric regulation? \\\ Allosteric inhibition is irreversible \\ Allosteric centre is the regulatory site of enzyme \\\ Allosteric modulator irreversibly binds with enzyme \\\ Allosteric modulator does not change conformation of active site \\\\ Which of the following does not belong to multienzymatic systems? \\\ Pyruvate dehydrogenase complex \\\ Alpha-ketoglutarate dehydrogenase complex \\ Glycolytic enzymatic system \\\ Enzymatic system of respiratory chain \\\\ Which coenzyme contains vitamin B6? \\ Piridoxalphosphate \\\ NAD+

\\\ CoA \\\ TPP \\\\ Which enzymes coenzyme is NAD+? \\\ Aminotransferases \\ Dehydrogenases \\\ Dehydratases \\\ Carboxylases \\\\ Which enzymes coenzyme is FAD? \\\ Proteases \\ Dehydrogenases \\\ Carboxylases \\\ Peptidases \\\\ Which enzymes coenzyme is FMN? \\\ Proteases \\\ Peptidases \\ Dehydrogenases \\\ Carboxylases \\\\ Substrate-H2 + NAD+ ------- substrate + NADH+ + H+ What is the enzyme of this reaction? \\\ FAD-dependent dehydrogenase \\\ Pyruvate dehydrogenase \\\ NADH-dehydrogenase \\ NAD-dependent dehydrogenase \\\\ Which enzymes coenzyme is NAD? \\\ Succinate dehydrogenase \\\ NADH dehydrogenase \\ Lactate dehydrogenase \\\ Pyruvate carboxilase \\\\ Which enzymes coenzyme is pyridoxal phosphate? \\\ Oxidoreductases \\ Transferases \\\ Hydrolases \\\ Isomerases Which enzymes coenzyme is ascorbate? \\\ Transferases \\\ Ligases \\\ Isomerases \\ Oxidoreductases \\\\ Which vitamin is part if pyridoxal phosphate? \\\ B1 \\\ B2 \\\ B3 \\ B6 \\\\ Which of the following belongs to oxidoreductases? \\\ Aldolase \\\ Glucokinase \\\ Citrate syntase \\ Succinate dehydrogenase \\\\ Which of the following does not belong to oxidoreductases?

\\\ isocitrate dehydrogenase \\\ malatedehydrogenase \\ lipoprotein lipase \\\ lactate dehydrogenase \\\\ What is oxidative-reductive coenzyme? \\ NAD+ \\\ Pyridoxal phosphate \\\ Coenzyme A \\\ Tiamin pyrophosphate \\\\ How the noncompetitive inhibitor influences on the kinetic indicators? \\\ Decreases Km without changes of Vmax \\\ Increases Km without changes of Vmax \\ Decreases Vmax without changes of Km \\\ Decreases both Vmax and Km \\\\ Which coenzyme is the donor of hydrogen ions in the process of pyruvate reduction? \\\ FADH2 \\\ FMNH2 \\ NADH \\\ NADPH \\\\ Which enzymes coenzyme is lipoic acid? \\\ Isocytreate dehydrogenase \\\ Malate dehydrogenase \\\ Citrate synthase \\ Alpha ketoglutarate dehydrogenase \\\\ Which does not belong to activation-transfer coenzymes? \\\ Biotin \\\ CoA \\\ Pyridoxal phosphate \\ FMN \\\\ Which does not belong to oxidation-reduction coenzymes? \\ Biotin \\\ NAD+ \\\ Vitamin C \\\ FMN \\\\ Which enzymes coenzyme is FAD? \\\ NADH dehydrogenase \\\ Malate dehydrogenase \\ Succinate dehydrogenase \\\ Pyruvate carboxilase \\\\ Which class of enzymes hexokinase belongs to? \\\ Lyases \\\ Hydrolases \\ Trasnferases \\\ Oxidoreductases \\\\ Which factor increases enzymes activity? \\\ Decrease in substrate concentration \\\ Increase in product concentration \\ Covalent modification

\\\ Gene repression \\\\ Which enzyme coenzyme is tiamin pyrophosphate? \\\ Succinate dehydrogenase \\ Pyruvate dehydrogenase \\\ Isocitrate dehydrogenase \\\ Fumarase \\\\ Which coenzyme participate in carboxylation reaction? \\\ FAD \\\ CoA \\ Biotin \\\ Vitamin C \\\\ Which of the following is not participating in oxidation-reduction processes? \\\ FAD \\ TPP \\\ NAD+ \\\ Vitamin E \\\\ Which class of enzymes glukokinase belongs to? \\\ Lyases \\\ Hydrolases \\ Transferases \\\ Oxidoreductases \\\\ Which enzymes activity increases in the blood during hepatitis? \\ Alanin aminotransferase \\\ Glytamate decarboxylase \\\ Creatine phosphokinase \\\ Aldolase \\\\ Which enzymes activity increases in the blood during myocardial infarction? \\ Aspartate aminotransferase \\\ Arginase \\\ Glutamate dehydrogenase \\\ Amylase \\\\ Which vitamins form oxidative-reductive coenzymes? \\ PP and B2 \\\ B6 and H \\\ K and A \\\ D and B1 \\\\ The enzymes are inhibited irreversibly by: \\ proteolytic cleavage \\\ Modulator protein \\\ Reaction product \\\ Allosteric modification \\\\ Which class of enzymes belong chymotripsin? \\\ Oxydoreductases \\ Hydrolases \\\ Transferases \\\ Lyases

TCA cycle
\\\\ Which of the following is not true of the citric acid cycle? \\ All enzymes of the cycle are located in the cytoplasm, except succinate dehydrogenase, which is bound to the inner mitochondrial membrane. \\\ Oxaloacetate is used as a substrate but is not consumed in the cycle. \\\ Succinate dehydrogenase channels electrons directly into the electron transfer chain. \\\ The Isocitrate dehydrogenase is subject of allosteric regulation by ATP and NADH. \\\\ Which of the following is an intermediate of the citric acid cycle? \\\ malonate \\\ alanine \\\ acyl-CoA \\ alpha-Ketoglutarate

\\\\ In mammals, which one is not occur during the citric acid cycle? \\\ formation of alpha-ketoglutarate. \\\ generation of NADH and FADH2. \\\ metabolism of acetate to carbon dioxide and water. \\ net synthesis of oxaloacetate from acetyl-CoA.
\\\\ What is the net production of conversion of 1 mol of acetyl-CoA to 2 mol of CO2 and CoA via the citric acid cycle? \\\ 1 mol of citrate. \\ 1 mol of FADH2. \\\ 1 mol of NADH. \\\ 7 mol of ATP. \\\\ Which one of the following is not associated with the oxidation of substrates by the citric acid cycle? \\\ CO2 production \\\ Flavin reduction \\\ Lipoic acid present in some of the enzyme systems \\ Pyridine nucleotide oxidation \\\\ The oxidative decarboxylation of alpha-ketoglutarate proceeds by means of multistep reactions in which all but one of the following cofactors is required. Which one is not required? \\ ATP \\\ Lipoic acid \\\ FAD \\\ Thiamine pyrophosphate \\\\ What reaction of the citric acid cycle is most similar to the pyruvate dehydrogenase complex-catalyzed conversion of pyruvate to acetyl-CoA? \\\ citrate to isocitrate. \\\ fumarate to malate. \\\ succinyl-CoA to succinate. \\ alpha-ketoglutarate to succinyl-CoA. \\\\ Which one of the following enzymatic activities would be decreased by thiamine deficiency? \\\ Isocitrate dehydrogenase \\\ Malate dehydrogenase \\\ Succinate dehydrogenase \\ alpha-Ketoglutarate dehydrogenase complex \\\\ Which reaction of the citric acid cycle is linked to the reduction of FADH2? \\\ isocitrate dehydrogenase. \\\ malate dehydrogenase. \\\ pyruvate dehydrogenase \\ succinate dehydrogenase. \\\\ Which of the following cofactors is required for the conversion of succinate to fumarate in the citric acid cycle?

\\\ Biotin \\ FAD \\\ NAD+ \\\ NADP+ \\\\ For what reaction is required flavin coenzyme In the citric acid cycle?:
\\\ condensation of acetyl-CoA and oxaloacetate. \\\ oxidation of fumarate. \\\ oxidation of isocitrate. \\ oxidation of succinate. \\\\ When decreases entry of acetyl-CoA into the citric acid cycle ? \\\ When [AMP] is high. \\\ When NADH is rapidly oxidized through the respiratory chain. \\ When the ratio of [ATP]/[ADP] is high. \\\ When the ratio of [NAD+]/[NADH] is high. \\\\ NAD+-specific isocitrate dehydrogenase and alpha-ketoglutarate dehydrogenase are two key regulatory enzymes of the citric acid cycle. What are the inhibitors of these enzymes? \\\ acetyl-CoA and fructose 6-phosphate. \\\ AMP and NAD+. \\\ ATP and NAD+. \\ ATP and NADH.

\\\\ For what of the following the inner mitochondrial membrane contains a transporter? \\\ NADH \\\ Acetyl CoA \\\ GTP \\ ATP \\\\ What is the substrate of isocitrate dehydrogenase? \\ isocitrate \\\ citrate \\\ alpha-ketoglutarate \\\ succinate \\\\ In what reaction NADH+H+ is produced? \\\ succinate dehydrogenase reaction \\ malate dehydrogenase reaction \\\ citrate synthase reaction \\\ aconitase \\\\ What is the major anaplerotic reaction of TCA cycle? \\\ alpha ketoglutarate dehydrogenase \\\ malate dehydrogenase \\\ enolase \\ pyruvate carboxylase \\\\ What is the substrate phosphorylation reaction in TCA cycle? \\\ formation of NADH+H \\\ formation of FADH2 \\ formation of GTP \\\ oxidation of isocitrate \\\\ What reactions product is GTP? \\\ succinate dehydrogenase reaction \\\ malate dehydrogenase reaction \\\ citrate synthase reaction \\ succinyl CoA synthetase reaction

\\\\ Which of the following is reversible reaction in TCA cycle? \\\ formation of citrate from acetyl Co A and oxaloacetate \\\ formation of alpha ketoglutarate from isocitrate \\ formation of oxaloacetate from malate \\\ formation of succinyl CoA from alpha ketoglutarate. \\\\ Which of the following is irreversible reaction in TCA cycle? \\ formation of citrate from acetyl CoA and oxaloacetate \\\ formation of malate from fumarate \\\ formation of oxaloacetate from malate \\\ formation of fumarate from succinate \\\\ What enzyme is allosterically inhibited by GTP and succinyl CoA in TCA cycle? \\\ malate dehydrogenase \\ alpha-ketoglutarate dehydrogenase \\\ citrate synthase \\\ pyruvate dehydrogenase \\\\ What reaction is catalyzed by pyruvate dehydrogenase complex? \\\ conversion of glucose into pyruvate \\ formation of acetyl-CoA from pyruvate \\\ formation of oxalocetate from pyruvate \\\ alanine conversion into pyruvate \\\\ What is characteristic for regulation of pyruvate dehydrogenase complex? \\\ phosphorylated form of the enzyme is active \\\ pyruvate dehydrogenase kinase dephosphorylates the enzyme \\ Ca2+ ions activate the enzyme \\\ ATP is activator of the enzyme \\\\ In which compartment of the cell is TCA cycle going? \\\ cytoplasm \\\ rough endoplasmic reticulum \\ mitochondria \\\ lysosoms \\\\ How many ATP is formed from NADH+H+ in electron transport chain? \\\ 2 \\ 2,5 \\\ 3 \\\ 1,5 \\\\ How many ATP is formed from FADH2 in electron transport chain? \\\ 2 \\\ 2,5 \\\ 3 \\ 1,5 \\\\ How many ATP is formed from TCA cycle with oxidative phosphorylation? \\\ 10 \\\ 1 \\ 9 \\\ 12 \\\\ How many ATP is formed from TCA cycle with substrate phosphorylation? \\\ 10

\\ 1 \\\ 9 \\\ 12 \\\\ How many ATP is produced in TCA cycle totally? \\ 10 \\\ 1 \\\ 9 \\\ 12 \\\\ Which of the following statements about the oxidative decarboxylation of pyruvate in aerobic conditions in animal cells is correct? \\ One of the products of the reactions of the pyruvate dehydrogenase complex is a thioester of acetate. \\\ The methyl (CH3) group is eliminated as CO2. \\\ The process occurs in the cytosolic compartment of the cell. \\\ The pyruvate dehydrogenase complex uses all of the following as cofactors: NAD+, lipoic acid, pyridoxal phosphate (PLP) and FAD. \\\\ The reaction of the citric acid cycle that produces an ATP equivalent (in the form of GTP) by substrate level phosphorylation is the conversion of: \\\ citrate to isocitrate. \\ fumarate to malate. \\\ malate to oxaloacetate. \\\ succinyl-CoA to succinate. \\\\ Which of the following reactions is anaplerotic (replenishes intermediate pools) for the citric acid cycle? \\\ Oxaloacetate + GTP ---- phosphoenolpyruvate + CO2 + GDP \\\ Malate + NAD+ ---- oxaloacetate + NADH + H+ \\\ Oxaloacetate + acetyl-CoA ----citrate + CoA \\ Pyruvate + HCO3+ ATP ----oxaloacetate + ADP + Pi + H+ \\\\ Which of the following intermediates is a substrate for the only membrane-associated enzymatic activity in the TCA cycle? \\\ citrate \\\ fumarate \\\ oxaloacetate \\ succinate \\\\ The ultimate acceptor of the electrons removed by oxidative decarboxylation in the TCA cycle is: \\\ NAD+ \\\ FAD \\\ ATP \\ O2 \\\\ Which of the following is NOT true of the citric acid cycle: \\ Most intermediates are activated by phosphorylation. \\\ It takes place in the mitochondria. \\\ Many of the reactions are oxidation-reduction reactions. \\\ It generates GTP by substrate -level phosphorylation \\\\ The citric acid cycle is amphibolic, that is it functions in both catabolism and anabolism. Certain intermediates are used to make other biomolecules including amino acids, nucleotide bases, fatty acids, and glucose. Several reactions replenish these intermediates for use in the citric acid cycle. These reactions are called: \\\ Parasynthetic \\\ Amphipathic \\ Anaplerotic

\\\ Amphibolic \\\\ How many NAD is generated in the TCA cycle? \\\ 1 \\\ 2 \\ 3 \\\ 4 \\\\ How many FADH2 is generated in the TCA cycle? \\ 1 \\\ 2 \\\ 3 \\\ 4 \\\\ How many ATP are produced after pyruvate oxidation (through the TCA cycle)? \\\ 10 ATP \\\ 30-32 ATP \\ 12,5 ATP \\\ 7,5 ATP \\\\ What is the functional group of NAD? \\\ Izoalloxazine ring \\ Nicotinamide \\\ Adenine \\\ Ribitole \\\\ How the regulations of metabolic ways generally go? \\\ By the alterations of intracellular metabolites concentration \\\ By the using of second messengers with peptide hormones \\\ By repression-induction of the genes \\ By all of the followings \\\\ What is the functional group of FAD? \\ Izoalloxazine ring \\\ Nicotinamide \\\ Adenosine residue \\\ Ribitole residue \\\\ What is characteristic for TCA cycle? \\\ Energetical effect of TCA cycle is 8 ATP \\ TCA cycle goes in aerobic conditions \\\ The final products are ammonia and water \\\ 3 ATP are produced in TCA cycle \\\\ What type of enzyme is Isocitrate dehydrogenase? \\\ It is regulatory enzyme of glycolysis \\\ It is FAD-dependent enzyme \\\ Belongs to transferases \\ It is decarboxylating dehydrogenase \\\\ How many macroergic bonds contain ATP? \\\ 1 \\ 2 \\\ 3 \\\ 4 \\\\ How many macroergic bonds contain ADP? \\ 1

\\\ 2 \\\ 3 \\\ 4 \\\\ How many macroergic bonds contain AMP? \\ 0 \\\ 1 \\\ 2 \\\ 3 \\\\ Which of the following is the marcroergic compound? \\\ AMP \\ 1,3 bisphosphoglycerate \\\ Malate \\\ Glucose-1-phosphate \\\\ Which of the following belongs to macroergic compounds? \\\ AMP \\ ADP \\\ GMP \\\ Glucose-1-phosphate \\\\ What is the delta-G of hydrolytic degradation of ATP? \\\ -5 kkal/mol \\ -7,3 kkal/mol \\\ -14,6 kkal/mol \\\ -20 kkal/mol \\\\ Which type of macroergic bond exists in acetyl-CoA? \\ Tyoesther \\\ Phospho anhydride \\\ Phospho enolic \\\ Phospho amide \\\\ What is the product of isocitrate dehydrogenase reaction? \\\ Malate \\\ Cytrate \\ Alpha-ketoglutarate \\\ Succinyl-CoA \\\\ What is the coenzyme of isocitrate dehydrogenase reaction? \\\ FAD \\\ TPP \\\ CoA \\ NAD+ \\\\ Which of the following is not necessary for alpha-ketoglutarate dehydrogenase reaction? \\\ FAD \\\ CoA \\\ NAD \\ ATP \\\\ What is energetical effect of alpha-ketoglutarate dehydrogenase reaction? \\\ 2 mole ATP \\ 2,5 mole ATP \\\ 3 mole ATP \\\ 6 mole ATP

\\\\ How many ATP is synthesized in the electron transport chain after isocitrate dehydrogenase reaction? \\\ 1 \\ 2,5 \\\ 3 \\\ 6,5 \\\\ How many ATP is synthesized in the electron transport chain after succinate dehydrogenase reaction? \\ 1,5 \\\ 2 \\\ 3,5 \\\ 6 \\\\ How many ATP is synthesized in the electron transport chain after malate dehydrogenase reaction? \\\ 1 \\ 2,5 \\\ 3 \\\ 6,5 \\\\ Which dehydrogenase is decarboxylating? \\ Isocitrate dehydrogenase \\\ malate dehydrogenase \\\ succinate dehydrogenase \\\ lactate dehydrogenase \\\\ Which of the following is not involved in the regulation of TCA cycle? \\\ Citrate synthatase \\\ Isocitrate dehydrogenase \\\ alpha-ketoglutarate dehydrogenase \\ aconitase \\\\ Which of the following is not regulatory enzyme of TCA cycle? \\\ Citrate synthatase \\ Malate dehydrogenase \\\ Isocitrate dehydrogenase \\\ alpha-ketoglutarate dehydrogenase \\\\ What are regulatory factors of TCA cycle? \\\ FADH2/FAD \\\ GTP/GDP \\ NADH/NAD \\\ ATP/GTP \\\\ Which compound is the substrate of TCA cycle? \\\ NADH \\\ NADPH \\ CH3CO-SCoA \\\ CH3COCOO\\\\ Succinyl-CoA+ GDP+H3PO4 -- Succinate+GTP+HS-CoA Which enzyme catalyzes this reaction? \\\ Fumarase \\ Succinate thyokinase \\\ Succinate dehydrogenase \\\ alpha-ketoglutarate dehydrogenase \\\\ Oxaloacetate+acetyl-CoA + H20 ---- citrate+HS-CoA \\\ Isocitrate dehydrogenase Which enzyme catalyzes this reaction?

\\\ Fumarase \\ Citrate synthase \\\ Succinate thyo kinase \\\\ Which compounds excess amount causes inhibition of oxidation of isocitrate? \\\ ADP \\\ NAD \\ NADH \\\ FAD \\\\ Which of the following activates the regulatory enzyme- isocitrate dehydrogenase? \\\ ATP \\\ NADH \\\ CoA \\ ADP \\\\ Isocitrate + NAD+ ---------- alpha-ketoglutarate+CO2+NADH+H+ Which enzyme catalyzes this reaction? \\ Isocitrate dehydrogenase \\\ malate dehydrogenase \\\ pyruvate dehydrogenase \\\ succinate thyokinase \\\\ Fumarate+H20 ------ malate \\\ malate dehydrogenase \\ fumarase \\\ alpha-ketoglutarate dehydrogenase \\\ succinate dehydrogenase Which enzyme catalyzes this reaction?

\\\\ Which vitamins deficiency influences on TCA cycle? \\\ Vitamin C \\ Thiamin \\\ Biotin \\\ Vitamin E \\\\ What is received in TCA cycle by substrate-level phosphorylation? \\\ UTP \\\ CTP \\\ NADH \\ GTP \\\\ Which compound inhibits the oxidation of alpha-ketoglutarate? \\\ ADP \\\ FAD \\ Succinyl-CoA \\\ TPP \\\\ In the TCA cycle the substrate of succinyl-CoA synthetase is: \\\ Alpha-ketoglutarate \\\ Fumarate \\\ Succinate \\ Succinyl-CoA \\\\ In which compartment of the cell the TCA cycle is localized? \\\ Cytosole \\\ Outer membrane if mitochondria \\ In the matrix of mitochondria

\\\ In the nucleus \\\\ Which conversion is impossible? \\\ Lactate ---- pyruvate \\\ Pyruvate ---- lactate \\ Acetyl-CoA ---- pyruvate \\\ Alanine ---- pyruvate \\\\ How many NADH is generated in the process of pyruvate decarboxylation? \\\ 3 \\\ 4 \\ 1 \\\ 5 \\\\ How many carboxylic groups exist in citrate? \\\ 1 \\\ 2 \\\ 4 \\ 3 \\\\ What is the irreversible reaction of TCA cycle? \\\ Malate dehydrogenase \\\ Fumarase \\ Alpha-ketoglutarate dehydrogenase \\\ aconitase \\\\ What compound is necessary for receiving the citrate from acetyl-CoA in the first reaction of TCA cycle? \\\ Isocitrate \\\ Malate \\ Oxaloacetate \\\ Succinate \\\\ What is the source of acetyl-CoA in the body? \\\ Glycogenesis \\\ Gluconeogenesis \\ Beta-oxidation of fatty acids \\\ Pentose phosphate pathway

CARBOHYDRATES METABOLISM
\\\\ What is the net gain of anaerobic conversion of 1 mol of glucose to 2 mol of lactate ? \\\ 1 mol of ATP. \\\ 1 mol of NADH. \\ 2 mol of ATP. \\\ 2 mol of NADH. \\\\ During strenuous exercise, the NADH formed in the glyceraldehyde 3-phosphate dehydrogenase reaction in skeletal muscle must be reoxidized to NAD+ if glycolysis is to continue. What is the most important reaction involved in the reoxidation of NADH ? \\\ glucose 6-phosphate ---- fructose 6-phosphate \\\ isocitrate ---- alpha-ketoglutarate \\\ oxaloacetate ----- malate \\ pyruvate ----- lactate \\\\ pyruvate (producing during glycolysis) in the erythrocyte further metabolizes into: \\\ CO2. \\\ glucose. \\\ hemoglobin. \\ lactate. \\\\ Which of these cofactors participates directly in most of the oxidation-reduction reactions in the fermentation of glucose to lactate? \\\ ADP \\\ ATP \\\ FAD/FADH2 \\ NAD+/NADH \\\\ Which of the following is not involved in steps of glycolysis between glyceraldehyde 3-phosphate and 3phosphoglycerate? \\\ catalysis by phosphoglycerate kinase. \\ oxidation of NADH to NAD+. \\\ the formation of 1,3-bisphosphoglycerate. \\\ utilization of Pi. \\\\ By what enzyme is catalyzed the first reaction in glycolysis that results in formation of an energy-rich compound (i.e., a compound whose hydrolysis has a highly negative delta G')? \\ glyceraldehyde 3-phosphate dehydrogenase. \\\ hexokinase. \\\ phosphofructokinase-1. \\\ triose phosphate isomerase. \\\\ What enzyme catalyzes glycogen conversion to monosaccharide units? \\\ glucokinase. \\\ glucose-6-phosphatase \\ glycogen phosphorylase. \\\ glycogen synthase. \\\\ Which of the following compounds cannot serve as the starting material for the synthesis of glucose via gluconeogenesis? \\ acetate \\\ glycerol \\\ lactate \\\ oxaloacetate

\\\\ What enzyme uses in both glycolysis and gluconeogenesis? \\ 3-phosphoglycerate kinase. \\\ glucose 6-phosphatase. \\\ hexokinase. \\\ phosphofructokinase-1. \\\\ Which one of the following statements about gluconeogenesis is false? \\\ For starting materials, it can use carbon skeletons derived from certain amino acids. \\\ It employs the enzyme glucose 6-phosphatase. \\\ It is one of the ways that mammals maintain normal blood glucose levels between meals. \\ It produces metabolic energy (ATP or GTP). \\\\ Which of the following enzymes is involved only in the flow of carbon from glucose to lactate (glycolysis) and not in the reversal of this flow (gluconeogenesis)? \\\ 3-phosphoglycerate kinase. \\\ aldolase. \\ phosphofructokinase-1. \\\ phosphoglucoisomerase. \\\\ What is characteristic feature of gluconeogenesis in humans? \\ can result in the conversion of protein into blood glucose. \\\ helps to reduce blood glucose after a carbohydrate-rich meal. \\\ is activated by the hormone insulin \\\ requires the enzyme hexokinase. \\\\ Which of the following substrates cannot contribute to net gluconeogenesis in mammalian liver? \\\ alanine \\\ glutamate \\ palmitate \\\ pyruvate \\\\ What is the main function of the pentose phosphate pathway? \\\ provide a mechanism for the utilization of the carbon skeletons of excess amino acids. \\\ supply energy. \\\ supply NADH. \\ supply pentoses and NADPH. \\\\ Which of the following enzymes acts in the pentose phosphate pathway? \\ 6-phosphogluconate dehydrogenase \\\ Glycogen phosphorylase \\\ Phosphofructokinase-1 \\\ Pyruvate kinase \\\\ What reaction is catalyzed by glycogen-branching enzyme? \\\ degradation of (alpha1 - 4) linkages in glycogen \\\ formation of (alpha1 - 4) linkages in glycogen. \\ formation of (alpha 1 - 6) linkages during glycogen synthesis. \\\ glycogen degradation in tree branches. \\\\ What is the characteristic for glycogenin: \\\ catalyzes the conversion of starch into glycogen. \\\ is the enzyme responsible for forming branches in glycogen. \\\ is the gene that encodes glycogen synthase. \\ is the primer on which new glycogen chains are initiated. \\\\ Which of the following is true of glycogen synthase? \\\ Activation of the enzyme involves a phosphorylation.

\\ It catalyzes addition of glucose residues to the nonreducing end of a glycogen chain by formation of (alpha1 4) bonds. \\\ It uses glucose-6-phosphate as donor of glucose units \\\ The conversion of an active to an inactive form of the enzyme is controlled by the concentration of cAMP. \\\\ What is characteristic for the enzyme glycogen phosphorylase? \\\ catalyzes a cleavage of (beta 1- 4) bonds. \\ catalyzes a hydrolytic cleavage of (alpha 1- 4) bonds. \\\ is a substrate for a kinase. \\\ uses glucose 6-phosphate as a substrate. \\\\ Gluconeogenesis must use bypass reactions to circumvent three reactions in the glycolytic pathway that are highly exergonic and essentially irreversible. Reactions carried out by which three of the enzymes listed must be bypassed in the gluconeogenic pathway? \\\ Phosphoglycerate kinase, glycogen syntase, aldolase \\ Hexokinase, Phosphofructokinase-1. Pyruvate kinase \\\ Lactate dehydrogenase, pyruvate dehydrogenase \\\ Triosephosphate isomerase, aldolase, pyruvate dehydrogenase \\\\ What is allosteric inhibitor of cellular isozymes of pyruvate kinase ? \\\ high concentration of AMP. \\ high concentration of ATP. \\\ high concentration of citrate. \\\ low concentration of acetyl-CoA. \\\\ Which of the following is true of glycogen synthesis and breakdown? \\ Phosphorylation activates the enzyme responsible for breakdown, and inactivates the synthetic enzyme. \\\ Synthesis is catalyzed by the same enzyme that catalyzes breakdown. \\\ The immediate product of glycogen breakdown is free glucose. \\\ Under normal circumstances, glycogen synthesis and glycogen breakdown occur simultaneously and at high rates. \\\\ Which combination of cofactors is involved in the conversion of pyruvate to acetyl-CoA? \\\ Biotin, FAD, and TPP \\\ NAD+, biotin, and TPP \\\ Pyridoxal phosphate, FAD, and lipoic acid \\ TPP, lipoic acid, and NAD+ \\\\ what is characteristic for glucokinase? \\\ acts in the conversion of liver glycogen to glucose 1-phosphate. \\\ converts fructose-6-phosphate to glucose-6-phosphate \\\ converts glucose 6-phosphate to fructose 6-phosphate. \\ is a hexokinase isozyme found in liver hepatocytes. \\\\ What is The Cori cycle? \\\ the conversion of lactate to pyruvate in skeletal muscle to drive glycogen synthesis. \\\ the interconversion between glycogen and glucose l-phosphate. \\ the production of lactate from glucose in peripheral tissues with the resynthesis of glucose from lactate in liver. \\\ the synthesis of urea in liver and degradation of urea to carbon dioxide and ammonia by bacteria in the gut. \\\\ When blood glucose is abnormally high, what is released from the pancreas? \\\ epinephrine. \\\ glucagon. \\ insulin. \\\ trypsin.

\\\\ When blood glucose is abnormally low, what is released from the pancreas? \\ glucagon. \\\ glucose. \\\ insulin. \\\ trypsin. \\\\ What type of linkages present between the glucose units of glycogen? \\\ alpha-1,4 linkages only \\ alpha-1,4 and alpha-1,6 linkages. \\\ beta-1,4 and beta-1,6 linkages. \\\ alpha-1,6 linkages only. \\\\ Which of the following statements is involved in the conversion of alpha-limit dextrins in intestinal carbohydrate digestion? \\\ they cannot be digested further and are passed in the feces. \\\ cleavage and anaerobic fermentation by bacteria in the lower gut. \\ hydrolysis by exosaccharidases in the glucoamylase complex. \\\ hydrolysis down to maltotriose units by the trehalase complex. \\\\ What is the best characterization of glucose transport into muscle? \\\ transport is elevated during starvation. \\\ sodium ions accompany glucose as it is taken up into muscle. \\\ uptake is mediated by GLUT-1 transporters in muscle. \\ transport is insulin-dependent. \\\\ What is the most important determinant of glucose homeostasis is normal individuals? \\\ insulin. \\ the ratio of glucagon to insulin. \\\ levels of thyroid hormone. \\\ plasma glucocorticoid concentrations. \\\\ Which enzyme uses inorganic phosphate as a substrate for phosphorylation during glycolysis? \\\ hexokinase \\\ phosphofructokinase-1 \\\ pyruvate kinase \\ glyceraldehyde-3-phosphate dehydrogenase \\\\ Which is the possible product of hexokinase? \\ glucose-6-phosphate \\\ 6-phosphogluconate \\\ fructose-1,6-bisphosphate \\\ glucose-1-phosphate \\\\ All of the following statements about glycogen are true EXCEPT \\\ A branched polymer made from glucose with a variable molecular weight between 10 and 100 million \\\ Contains chains of glucosyl units linked by alpha-1,4 bonds with alpha-1,6 branches \\ The many glucosyl residues that are attached to the fatty acid glycogennin are designated as the reducing ends of the polymer \\\ The enzymes involved in glycogen synthesis and degradation are bound to the surface of the glycogen particles. \\\\ Which of the following statements about glycogen and its catabolism is NOT true? \\\ The function of glycogen in liver is to supply glucose to the blood when needed \\\ The function of glycogen in muscle is to supply energy for muscle contraction \\\ When glycogenolysis in active in muscle, glycolysis is usually active \\ Muscle, but not liver, has the enzyme glucose-6-phosphatase

\\\\ What processes are activated in the liver by glucagon ? \\\ Glycolysis and glycogenolysis \\\ Glycolysis and glycogen synthesis \\ Gluconeogenesis and glycogenolysis \\\ Gluconeogenesis and glycogen synthasis \\\\ When glycogen is synthesized in the liver and muscle, all of the following are true EXCEPT \\\ Glucose enters the liver cell and is phosphorylated by glucokinase to become glucose-6-phosphate \\\ The isomerase, phosphoglucomutase, converts glucose-6-P to glucose-1-P \\\ Glucose is transferred from UDP-glucose to a glycogen molecule by glycogen synthase \\ Glycogen synthase is activated by glucagon and inhibited by insulin \\\\ When glycogen is degraded (catabolized), all of the following occur in both liver and muscle EXCEPT \\\ Active glycogen phosphorylase catalyzes the phosphorolysis of alpha-1,4-glucosidic bonds in glycogen \\\ Debranching enzyme catalyzes the hydrolysis of alpha-1,6-glucosidic bonds in glycogen \\\ Glucose-1-P is converted to glucose-6-P by phosphoglucomutase \\ glycogen degradation process initiates after taking high carbohydrate meal. \\\\ All of the following statements about glycogen storage diseases are true EXCEPT \\ The pathology is usually associated with the accumulation of too much glycogen in all cells of the body \\\ In McArdles disease the defect is in glycogen phosphorylase \\\ In McArdles disease, not enough energy can be supplied by anaerobic glycolysis during vigorous exercise \\\ In Von Gierkes disease, glucose-6-phosphatase is deficient \\\\ Which statement about glucose and liver is NOT true? \\\ A high carbohydrate meal will raise the level of blood glucose \\\ Since the entrance of glucose into liver cells is passive, high blood glucose causes high glucose concentrations in the cytosol \\\ Glucokinase will convert glucose to glucose-6-P when glucose concentrations are high \\ High glucose concentrations will activate gluconeogenesis and glycogenolysis in liver \\\\ All of the following statements concerning glycogen synthesis and glycogenolysis are true EXCEPT \\\ High blood glucose and high insulin will activate glycogen synthesis and inhibit glycogen phosphorylase \\\ A low insulin to glucagon ratio will activate glycogen phosphorylase and inhibit glycogen synthase \\\ Exercise will increase catecholamines, activate glycogen phosphorylase and inhibit glycogen synthase \\ Taking this test will inhibit glycogen phosphorylase and activate glycogen synthase \\\\ All of the following statements about blood glucose and muscle are true EXCEPT \\\ Increased blood glucose will increase insulin and cause more Glut-4 (glucose transporter-4) to enter the cell membrane \\\ Striated muscles do not have glucagon receptors and so glucagon does not have an effect on muscle glycogen phosphorylase \\ Catecholamines cause more insulin to be released during exercise \\\ Catecholamines activate the cAMP cascade and glycogen phosphorylase but inhibit glycogen synthase \\\\ The insulin to glucagon ratio is low as it usually is in both Type I and Type II diabetics. What would you expect during and immediately after a high carbohydrate meal? \\\ The storage of glucose in muscle to be less than normal for type 2 patients but normal for type 1 patients \\\ The storage of glucose in muscle to be less than normal for type 1 patients but normal for type 2 patients \\ The storage of glucose in muscle and liver to be less than normal \\\ The storage of glucose in muscle and liver to be greater than normal \\\\ At the same time that epinephrine activates the beta receptors it also binds to alpha-1 receptors. When the alpha-1 is activated, all of the following result EXCEPT \\\ The inactivation of glycogen synthase \\ The activation of glycogen phosphorylase by cAMP pathway \\\ The activation of phosphorylase kinase by Ca++-calmodulin \\\ The activation of phospholipase C and the production of diacylglycerol

\\\\ Which of the following will result in activation of glycogen synthase? \\\ Increased concentrations of AMP from contraction of muscle \\\ Increased cytosolic [Ca++] \\ Increased protein phosphatase \\\ Increased activity of phosphorylase kinase \\\\ All of the following statements about the pentose phosphate pathway are true EXCEPT \\\ Its two functions are to produce NADPH and ribose-5-P \\\ It uses glucose-6-P as a substrate when producing NADPH and CO2 \\\ Glucose-6-phosphate dehydrogenase is the control enzyme and it is regulated by the NADPH concentration of the cell \\ It is found in the mitochondria of liver, muscle and brain but is absent from most other tissues of the body \\\\ If a cell only needs to synthesize ribose-5-P and not NADPH, the enzymes of the hexose monophosphate shunt what of the following substrates would use from glycolysis? \\\ Fructose-6-phosphate and NADPH \\\ 3-Phosphoglycerate and fructose-6-phosphate \\ Glyceraldehyde-3-phosphate and fructose-6-phosphate \\\ Glucose-1-phosphate and lactate \\\\ What is the main function of NADPH in most cells? \\\ it serves as a substrate for the electron transport chain \\\ it produces ribose-5-P from glyceraldehyde-3-P and fructose-6-P \\ it serves as a agent in detoxification reactions \\\ it serves as an oxidizing agent in reductive biosynthesis \\\\ You have a patient that has hemolytic anemia as a result of eating fava beans. What enzyme deficiency would the patient have? \\\ Glucose-6-P phosphatase \\\ Phospho fructo isomerase; \\ Glucose-6-P dehydrogenase \\\ 6-Phosphogluconate dehydrogenase; \\\\ You have a patient that has hemolytic anemia as a result of eating fava beans. What compound(s) deficiency would the patient have? \\\ ribose-5-P and ribulose-5-P \\ NADPH \\\ NADH \\\ FAD \\\\ Your patient has glucose-6-phosphate dehydrogenase deficiency as a result of taking a sulfa drug. He has never been diagnosed with this disease before. He suffers from hemolysis while his white blood cells look OK. What the following statements is true? \\\ He has no fairly common genetic disease \\ This is probably the first time his system has been challenged with a lot of free radicals (reactive oxygen species) \\\ He has severe deficiency of glucose-phosphate dehydrogenase activity - more than most people \\\ Most cells were lysed because the PPP is only way for producing NADPH \\\\ which of the following is a common intermediate in the conversion of glycerol and lactate to glucose? \\\ Pyruvate \\\ Oxaloacetate \\ Glucose 6-phosphate \\\ Phosphoenolpyruvate \\\\ A patient presented with a bacterial infection that produced an endotoxin that inhibits

phosphoenolpyruvate carboxykinase. In this patient, then, under these conditions, glucose production from which of the following precursors would be inhibited? \\ Alanine \\\ Glycerol \\\ Phosphoenolpyruvate \\\ Galactose \\\\ The enzyme phosphofructokinase-2/fructose-2,6-bisphosphatase plays a key role in the control of glycolysis exerted by the insulin to glucagon ratio. If the insulin to glucagon ratio increases, what would happen to phosphofructokinase-2/fructose-2,6-bisphosphatase?: \\\ Become more phosphorylated and phosphofructokinase-2 would become more active \\\ Become less phosphorylated and phosphofructokinase-2 would become less active \\\ Become more phosphorylated and phosphofructokinase-2 would become more active \\ Become less phosphorylated and phosphofructokinase-2 would become more active \\\\ What would happen with liver enzyme pyruvate kinase during a fast? \\\ Becomes more active and more phosphorylated \\\ Becomes more active and less phosphorylated \\ Becomes less active and more phosphorylated \\\ Become less active and less phosphorylated \\\\ A patient takes sulfa drugs for the first time and develops hemolytic anemia. The other blood cells appear normal. What is the best explanation of this? \\\ The red blood cells have more active glucose-6-phosphate dehydrogenase \\\ The red blood cells have more glutathione reductase activity \\\ The red blood cells have more NADH in their cytosol \\ The other cells have more active glucose-6-phosphate dehydrogenase activity \\\\ After ingestion of a high carbohydrate meal, which of the following is true? \\\ Glucose uptake and utilization by adipose tissue is impaired. \\\ Glycogen degradation is increased \\\ Glycolysis in the liver is decreased \\ Glucose uptake by the brain is increased \\\\ What enzyme is activated with increased concentration of acetyl-CoA in the liver cell during gluconeogenesis? \\ Mitochondrial pyruvate carboxylase \\\ Cytosolic phosphoenolpyruvate carboxykinase \\\ Cytosolic pyruvate carboxylase \\\ Cytosolic pyruvate kinase \\\\ When pyruvate carboxylase and phosphoenolpyruvate carboxykinase are active, by what is prevented futile cycling? \\\ Phosphorylation and activation of pyruvate kinase by insulin via cAMP \\\ Dephosphorylation and inactivation of protein kinase by glucagon via cAMP \\\ Phosphorylation and activation of protein kinase by insulin via cAMP \\ Phosphorylation and inactivation of pyruvate kinase by glucagon via cAMP \\\\ What process is activated with increase in the insulin/glucagon ratio? \\ The dephosphorylation of pyruvate kinase and an increase in the conversion of phosphoenolpyruvate to pyruvate \\\ The phosphorylation of pyruvate kinase and an increase in the conversion of phosphoenolpyruvate to pyruvate \\\ The phosphorylation of pyruvate kinase and an decrease in the conversion of phosphoenolpyruvate to pyruvate \\\ The dephosphorylation of pyruvate kinase and an decrease in the conversion of phosphoenolpyruvate to pyruvate

\\\\ What is characteristic for the hexose monophosphate shunt? \\\ Found mostly in the mitochondria of muscle cells \\\ Found mostly in the cytosol of cells that rely heavily upon anaerobic glycolysis \\\ Found in the mitochondria of all cells \\ Found in the cytosol of all cells \\\\ If a cell needs to make exactly twice the amount of NADPH as ribose-5-phosphate, which glycolytic substrate(s) would be used? \\\ Glucose-6-phosphate and fructose-6-phosphate \\ Glucose-6-phosphate \\\ Fructose-6-phosphate and glyceraldehyde-3-phosphate \\\ Glucose-6-phosphate and dihydroxyacetone phosphate \\\\ A person fasts for 12 hours and then eats a high protein meal. About 45 minutes later their blood is analyzed. What would be blood indicators before ingestion? \\ Insulin will be higher. Glucagon will be higher. Liver gluconeogenesis will be active. \\\ Insulin will be lower. Glucagon will be higher. Liver glycolysis will be active. \\\ Insulin will be lower. Glucagon will be higher. Liver gluconeogenesis will be active. \\\ Insulin will be higher. Glucagon will be lower. Liver gluconeogenesis will be active. \\\\ Regarding dietary glucose entering the liver following a high caloric, high carbohydrate meal which of the following statements is NOT true: \\\ Some glucose is converted to glycerol phosphate \\ Some glucose is released from the liver glycogen \\\ Some glucose is used via aerobic glycolysis \\\ Some glucose is used for fatty acid synthesis \\\\ When going from the fed state to the fasted state, the activities of the glycolytic and gluconeogenic pathways are changed. Which one is the part of the mechanism for this change? \\\ An increase in the activity of 3',5'-phosphosdiesterase \\\ An increase in the activity of phosphofructokinase-2 \\ An increase in the activity of fructose-2,6-bisphosphatase \\\ A decrease in the activity of protein kinase \\\\ You meet your patient in the emergency room. She forgot that she took her first shot of insulin last night and so she took a second shot. She is now in a hypoglycemic coma. Which of the following is contributing to the low blood sugar? \\\ Insulin has activated the cyclic AMP cascade in the liver \\\ Insulin has activated the cyclic AMP cascade in adipose tissue \\\ Insulin has caused an increase in ketone bodies synthesis in the liver \\ Insulin has inhibited fructose-1,6-biphosphatase in the liver \\\\ By what of the following Ca2+ increases glycogenolysis? \\ activating phosphorylase kinase b, even in the absence of cAMP \\\ binding to phosphorylase b. \\\ Inhibiting phosphoprotein phosphatase \\\ Activating phosphoprotein phosphatase \\\\ What is the reason of von Gierkes disease? \\\ glucose-6-phosphate dehydrogenase deficiency \\ glucose-6-phosphatase deficiency \\\ fructose 1,6 bisphosphatase deficiency \\\ pyruvate kinase deficiency \\\\ In comparison with the resting state, actively contracting human muscle tissue has a: \\\ higher concentration of ATP.

\\ higher rate of lactate formation. \\\ lower consumption of glucose. \\\ lower ratio of NADH to NAD+. \\\\ Which of the following statements is incorrect? \\\ Aerobically, oxidative decarboxylation of pyruvate forms acetate that enters the citric acid cycle. \\\ In anaerobic muscle, pyruvate is converted to lactate. \\\ Reduction of pyruvate to lactate regenerates a cofactor essential for glycolysis. \\ Under anaerobic conditions pyruvate does not form because glycolysis does not occur. \\\\ In humans, gluconeogenesis: \\ can result in the conversion of protein into blood glucose. \\\ helps to reduce blood glucose after a carbohydrate-rich meal. \\\ is activated by the hormone insulin \\\ requires the enzyme hexokinase. \\\\ A 13 year-old patient with Type I diabetes mellitus, home alone, takes an insulin injection before lunch but then he gets concentrated in a videogame he is playing and does not eat. Approximately 3 hours later, he becomes sweat, shaky and confused. These symptoms have appeared as a consequence of: \\\ increased glucagon release from the pancreas \\\ increased insulin release from the pancreas \\\ high sugar levels in blood \\ low sugar levels in blood \\\\ Which of the following sentences explain better the role of fructose 2,6-bisphosphate in glycolysis? \\\ It antagonizes phosphofructokinase-1 \\\ It activates covalently aldolase \\\ It inhibits allosterically hexokinase \\ It allosterically activates phosphofructokinase-1 \\\\ Which of the following compounds is an allosteric activator of glycogen phosphorylase b in muscle? \\ AMP \\\ ATP \\\ Glucose \\\ GTP \\\\ If a cell has high levels of ATP, which of the following enzymes will be negatively affected? \\\ Fructose 1, 6-bisphosphatase \\\ Hexokinase \\ Phosphofructokinase and pyruvate kinase \\\ Glycogen phosphorylase and glycogen synthase \\\\ The Cori cycle describes the release of lactate from active muscle into the bloodstream and its subsequent uptake by the liver. The lactate in the liver is then used for: \\\ Glycogen synthesis \\\ ATP phosphorylation \\\ Phosphocreatine synthesis \\ Glucose synthesis \\\\ Why untreated diabetics develop ketosis? \\\ The cells can utilize more amount of glucose, than they need. \\\ They have increased glycogen levels. \\ The cells cannot utilize glucose. \\\ Fatty acid synthesis is increased \\\\ What is characteristic for hexokinase? \\ High affinity for glucose

\\\ Low affinity for glucose \\\ It has very high Km \\\ Is induced by insulin \\\\ What is characteristic for glucokinase? \\\ High affinity for glucose \\ Low affinity for glucose \\\ It has low Km \\\ Is inhibited by product of reaction \\\\ Which enzyme decreases cAMP level? \\\ adenylate cyclase \\ phosphodiesterase \\\ protein kinase \\\ phospho protein phosphatase \\\\ Which hormone activates phosphodiestherase? \\\ glucagon \\ insulin \\\ adrenalin \\\ growth factor \\\\ In which tissues is glucose transport depend on the insulin? \\\ in the lens \\\ in the liver \\ in the muscular tissue \\\ in the erythrocyte \\\\ In which tissues is glucose transport depend on the insulin? \\\ in the lens \\\ in the liver \\ in the adipose tissue \\\ in the erythrocyte \\\\ In which compartment pyruvate derived from aerobic degradation of glucose is burnt? \\\ Golji complex \\\ endoplasmic reticulum \\ mitochondria \\\ lisosoms \\\\ Which compound is inhibitor of pyruvate dehydrogenase \\\ FAD \\ Acetyl-CoA \\\ Ca2+ \\\ NAD \\\\ NADH+H+ produced in the glyceraldehyde dehydrogenase reaction during anaerobic conditions is used in: \\\ Respiratory chain \\ Pyruvate reduction \\\ pentose phosphate pathway \\\ glutathione oxidation reaction \\\\ Which is the end product of glucose degradation during aerobic conditions? \\\ lactate \\\ pyruvate \\\ acetyl-CoA

\\ CO2 and H20 \\\\ In which tissue hexokinase does not catalyze phosphorylation of glucose? \\ liver \\\ brain \\\ muscular tissue \\\ erythrocytes \\\\ In which tissue glucokinase catalyzes phosphorylation of glucose? \\ pancreas \\\ brain \\\ muscular tissue \\\ erythrocytes \\\\ Which compound provides activation of gluconeogenesis? \\\ NAD \\ acetyl-CoA \\\ FAD \\\ CoA \\\\ How glucagon and epinephrine prevent decrease in glucose level in the blood? \\ activation of gluconeogenesis \\\ inhibition of gluconeogenesis \\\ enhancing lipogenesis \\\ inhancing glycogenesis in the liver \\\\ How glucagon and epinephrine provide hyperglycemia? \\\ activation of lipogenesis \\\ activation of glycogenesis \\\ inhibition of gluconeogenesis \\ activation of gluconeogenesis \\\\ Which compound is involved in the regulation of glycolysis and gluconeogenesis? \\\ fructose-1,6 bisphosphate \\ fructose-2,6 bisphosphate \\\ 1,3 bisphosphoglycerate \\\ glyceraldehyde-3-phosphate \\\\ Which hormone activates glycogene phosphorylase? \\\ Insulin \\\ Tyroxin \\\ Estrogens \\ Glucagon \\\\ Which enzyme is involved in the oxidative decarboxylation of pyruvate? \\\ pyruvate carboxylase \\ pyruvate dehydrogenase \\\ alpha-ketoglutarate dehydrogenase \\\ acetyl-CoA carboxylase \\\\ How insulin prevents increase in glucose level in the blood? \\\ by stimulation of gluconeogenesis \\ by inhibition of gluconeogenesis \\\ by inhibition of lipogenesis \\\ by inhancing glycogenolysis in the liver \\\\ Which is the substrate of enolase?

\\\ citrate \\\ aconitate \\\ phosphoenolpyruvate \\ 2-phosphoglycerate \\\\ How the absorption of monosaccharides proceeds? \\\ Fructose is absorbed with the passive diffusion in the enterocytes \\\ specific transporter, exist on the lumen site of enterocytes plasma membrane, provides glucose absorption with passive facilitated diffusion \\\ symport mechanism is characteristic for active absorption of glucose. \\ all answers are correct \\\\ During the gluconeogenic conversion of pyruvate into glucose in the liver, all of the following are involved EXCEPT \\\ pyruvate carboxylase \\ phosphoenolpyruvate carboxylase \\\ phosphoenolpyruvate carboxykinase \\\ glucose 6-phosphatase

\\\\ In the cells stimulated by glucagon, the cAMP initiates an enzymatic cascade that begins with the activation of protein kinase A (PKA) and whose main result is: \\ increased glycogenolysis \\\ increased glycogenesis \\\ increased glycolysis \\\ decreased gluconeogenesis \\\\ In the cells stimulated by glucagon, the cAMP initiates an enzymatic cascade that begins with the activation of protein kinase A (PKA) and whose main results is: \\\ Decreased glycogenolysis \\ Decreased glycogenesis \\\ Decreased fatty acid mobilization \\\ Decreased ketogenesis \\\\ In the cells stimulated by glucagon, the cAMP initiates an enzymatic cascade that begins with the activation of protein kinase A (PKA) and whose main results is: \\\ Decreased glycogenesis \\ Decreased glycolysis \\\ decreased gluconeogenesis \\\ decreased fatty acid mobilization \\\\ In the cells stimulated by glucagon, the cAMP initiates an enzymatic cascade that begins with the activation of protein kinase A (PKA) and whose main results is: \\\ Decreased glycogenolysis \\\ increased glycogenesis \\\ increased glycolysis \\ Increased gluconeogenesis \\\\ In the cells stimulated by glucagon, the cAMP initiates an enzymatic cascade that begins with the activation of protein kinase A (PKA) and whose main results is: \\\ Increased glycolysis \\\ Decreased gluconeogenesis \\ Increased fatty acid movilization \\\ Decreased ketogenesis \\\\ In the cells stimulated by glucagon, the cAMP initiates an enzymatic cascade that begins with the

activation of protein kinase A (PKA) and whose main results is: \\\ increased glycogenesis \\\ increased glycolysis \\\ decreased gluconeogenesis \\\ Increased ketogenesis \\\\ which enzyme is deficient during Pompes disease (glycogen storage disease type II)? \\\ glucose-6-phosphatase \\ alpha 1,4-glucosidase \\\ branching enzyme \\\ glukokinase \\\\ which enzyme is deficient during McArdles disease (glycogen storage disease type V)? \\\ glucose-6-phosphatase \\ muscle phosphorylase \\\ branching enzyme \\\ liver phosphorylase \\\\ which enzyme is deficient during Coris (glycogen storage disease type III) disease? \\\ glucose-6-phosphatase \\ debranching enzyme \\\ branching enzyme \\\ liver phosphorylase

FATTY ACIDS
\\\\ What is the action of lipoprotein lipase? \\ hydrolysis of triacylglycerols of plasma lipoproteins to supply fatty acids to various tissues. \\\ intestinal uptake of dietary fat. \\\ intracellular lipid breakdown of lipoproteins. \\\ lipoprotein breakdown to supply needed amino acids. \\\\ What is true about free fatty acids in the bloodstream? \\\ bound to hemoglobin. \\ carried by the protein serum albumin. \\\ freely soluble in the aqueous phase of the blood. \\\ nonexistent; the blood does not contain free fatty acids. \\\\ What is the role of hormone-sensitive triacylglycerol lipase? \\\ hydrolyze lipids stored in the liver. \\ hydrolyze triacylglycerols stored in adipose tissue. \\\ synthesize lipids in adipose tissue. \\\ synthesize triacylglycerols in the liver. \\\\ What is required for transporting of fatty acids from the cytoplasm to the mitochondrial matrix? \\ ATP, carnitine, and coenzyme A. \\\ ATP, carnitine, and pyruvate dehydrogenase. \\\ ATP, coenzyme A, and hexokinase. \\\ ATP, coenzyme A, and pyruvate dehydrogenase. \\\\ Fatty acids are activated to acyl-CoAs and the acyl group is further transferred to carnitine. Why is this process so important? \\ acyl-carnitines readily cross the mitochondrial inner membrane, but acyl-CoAs do not. \\\ acyl-CoAs easily cross the mitochondrial membrane, but the fatty acids themselves will not. \\\ carnitine is required to oxidize NAD+ to NADH. \\\ fatty acids cannot be oxidized by FAD unless they are in the acyl-carnitine form. \\\\ Which of these is able to cross the inner mitochondrial membrane? \\\ AcetylCoA \\ Fatty acylcarnitine \\\ Fatty acylCoA \\\ MalonylCoA \\\\ What is the correct order of function of the following enzymes of beta- oxidation? \\\ beta-hydroxyacyl-CoA dehydrogenase, thiolase, enoyl-CoA hydratase, acyl-CoA dehydrogenase \\\ Thiolase, enoyl-CoA hydratase, acyl-CoA dehydrogenase, beta-hydroxyacyl-CoA dehydrogenase \\ acyl-CoA dehydrogenase, enoyl-CoA hydratase, beta-hydroxyacyl-CoA dehydrogenase, thiolase \\\ Acyl-CoA dehydrogenase , thiolase, enoyl-CoA hydratase, beta-hydroxyacyl-CoA dehydrogenase, \\\\ If the 16-carbon saturated fatty acid palmitate is oxidized completely to carbon dioxide and water (via the betaoxidation pathway and the citric acid cycle), and all of the energy-conserving products are used to drive ATP synthesis in the mitochondrion, What would be net yield of ATP per molecule of palmitate? \\\ 3. \\\ 25. \\ 108. \\\ 1,000. \\\\ Saturated fatty acids are degraded by the stepwise reactions of beta-oxidation, producing acetyl-CoA. Under aerobic conditions, how many ATP molecules would be produced as a consequence of removal of each acetyl-CoA? \\\ 2 \\\ 3 \\ 4 \\\ 5

\\\\ Which of the following is (are) true of the oxidation of 1 mol of palmitate (a 16-carbon saturated fatty acid; 16:0) by the beta-oxidation pathway, beginning with the free fatty acid in the cytoplasm? \\ Activation of the free fatty acid requires the equivalent of two ATPs. \\\ Carnitine functions as an electron acceptor. \\\ 8 mol of FADH2 are formed. \\\ There is no direct involvement of NAD+. \\\\ Which of the following is (are) true of the oxidation of 1 mol of palmitate (a 16-carbon saturated fatty acid; 16:0) by the beta-oxidation pathway, beginning with the free fatty acid in the cytoplasm? \\\ Activation of the free fatty acid requires the equivalent of three ATPs. \\\ Carnitine functions as an electron acceptor. \\ 8 mol of acetyl-CoA are formed. \\\ There is no direct involvement of NAD+. \\\\ Which of the following statements apply to the beta- oxidation of fatty acids? \\\ The process takes place in the cytosol of mammalian cells. \\\ Before oxidation, fatty acids must be converted to their CoA derivatives. \\\ Carbon atoms are removed from the acyl chain one at a time. \\\ NADP+ is the electron acceptor. \\ The products of beta oxidation can directly enter the citric acid cycle for further oxidation. \\\\ Which of the following statements concerning the beta-oxidation of fatty acids is true? \\\ About 1,200 ATP molecules are ultimately produced per 20-carbon fatty acid oxidized. \\\ One FADH2 and two NADH are produced for each acetyl-CoA. \\ The free fatty acid must be converted to a thioester before the process of beta-oxidation commences. \\\ Two NADH are produced for each acetyl-CoA. \\\\ Which compound is an intermediate of the beta- oxidation of fatty acids? \\\ CH3(CH2)20COCOOH \\\ CH3CH2COCH2COOPO32 \\\ CH3CH2COCH2OH \\ CH3COCH2COSCoA \\\\ What is the net formation of conversion of palmitoyl-CoA (16:0) to myristoyl-CoA (14:0) and 1 mol of acetylCoA by the beta-oxidation pathway? \\ 1 FADH2 and 1 NADH. \\\ 1 FADH2, 1 NADH, and 1 ATP. \\\ 2 FADH2 and 2 NADH. \\\ 2 FADH2, 2 NADH, and 1 ATP. \\\\ Which of the following is not true regarding the oxidation of 1 mol palmitate (16:0) by the beta- oxidation pathway? \\\ 1 mol of ATP is needed. \\\ 8 mol of acetyl-CoA are formed. \\ 8 mol of FADH2 are formed. \\\ The reactions occur in the mitochondria. \\\\ Which of the following is true of the beta- oxidation of long-chain fatty acids? \\\ The enzyme complex that catalyzes the reaction contains biotin. \\ FADH2 and NADH serve as electron carriers. \\\ Oxidation of an 18-carbon fatty acid produces six molecules of propionyl-CoA. \\\ NADPH serves as an electron carrier. \\\\ Which of the following is true of the beta- oxidation of long-chain fatty acids? \\\ The enzyme complex that catalyzes the reaction contains biotin. \\\ FMN serves as an electron carrier. \\\ Oxidation of an 18-carbon fatty acid produces six molecules of propionyl-CoA. \\ Oxidation of a 15-carbon fatty acid produces at least one propionyl-CoA.

\\\\ The carbon atoms from a fatty acid with an odd number of carbons will enter the citric acid cycle as acetyl-CoA and: \\\ butyrate. \\\ citrate. \\ succinyl-CoA. \\\\ alpha-ketoglutarate. \\\\ Ketone bodies are formed in the liver. By which form are they transported to the extrahepatic tissues? \\\ acetoacetyl-CoA. \\\ acetone. \\ beta-hydroxybutyric acid. \\\ lactic acid. \\\\ What is the major site of formation of acetoacetate from fatty acids? \\\ adipose tissue. \\\ intestinal mucosa. \\ liver. \\\ muscle. \\\\ Which of the following is required in the synthesis of fatty acids? \\ NADPH \\\ pyridoxalphosphate \\\ lipoic acid \\\ FADH2 \\\\ Which of the following is true of the reaction producing malonyl-CoA during fatty acid synthesis? \\\ It is stimulated by cholesterol. \\\ It requires acyl carrier protein (ACP). \\ It requires CO2 (or bicarbonate) , ATP and biotin. \\\ Three moles of ATP is converted to ADP + Pi for each malonyl-CoA synthesized. \\\\ What is the rate-limiting step in fatty acid synthesis? \\\ condensation of acetyl-CoA and malonyl-CoA. \\\ formation of acetyl-CoA from acetate. \\\ formation of malonyl-CoA from malonate and coenzyme A. \\ the reaction catalyzed by acetyl-CoA carboxylase. \\\\ Which of these statements about triacylglycerol synthesis is correct? \\\ Humans can store more energy in glycogen than in triacylglycerols. \\ Insulin stimulates conversion of dietary carbohydrate into triacylglycerols. \\\ It is not a hormone-sensitive process. \\\ Mammals are unable to convert carbohydrates into triacylglycerols. \\\\ From which compound is synthesized cholesterol? \\ acetyl-CoA. \\\ choline. \\\ lipoic acid. \\\ malate. \\\\ What is the 30-carbon precursor of the steroid nucleus? \\\ farnesyl pyrophosphate. \\\ geranyl pyrophosphate. \\\ lysolecithin. \\ squalene. \\\\ Which of the following is derived from a sterol? \\ Bile salts \\\ Gangliosides \\\ Geraniol \\\ Phosphatidylglycerol

\\\\ What is the largest energy store in a well-nourished human? \\\ ATP in all tissues. \\\ blood glucose. \\\ muscle glycogen. \\ triacylglycerols in adipose tissue.

\\\\ What are bile salts? \\ amphipathic cholesterol analogs with detergent properties \\\ charged phospholipids \\\ hydrolyzed forms of triacylglycerols \\\ esterified cholesterols \\\\ What is the lipoprotein with the highest ratio of protein:lipid? \\\ chylomicrons \\\ VLDL \\\ LDL \\ HDL \\\\ What does the core of a typical lipoprotein primarily consist of ? \\\ apoB-100 \\\ phospholipid \\\ cholesterol \\ triacylglycerols and cholesterol esters \\\\ What lipoprotein delivers dietary cholesterol to the liver? \\\ IDL \\ chylomicron remnants \\\ LDL \\\ HDL \\\\ Which lipoprotein accumulates in the bloodstream of patients with familial hypercholesterolemia? \\ LDL \\\ IDL \\\ HDL \\\ chylomicron \\\\ What is the intracellular location of cholesterol biosynthesis? \\\ mitochondrial matrix \\ cytoplasm \\\ golgi apparatus \\\ nucleus \\\\ \\ \\\ \\\ \\\ Where can cholesterol be synthesized? in most mammalian cells in brain under anaerobic conditions only in adipose tissue only in the liver

\\\\ In humans, by which form is cholesterol degraded and excreted? \\ bile salts. \\\ CO2 and H2O. \\\ bilirubin. \\\ ketone bodies. \\\\ What is the key regulatory step in cholesterol biosynthesis? \\ HMG-CoA reductase \\\ mevalonate kinase

\\\ prenyl transferase \\\ phophomevalonate kinase \\\\ Malonyl ACP, a three-carbon unit, is the starting material for fatty-acid biosynthesis. However, the product fatty acid, at least in mammals, contains an even number of carbons. How is this accomplished? \\\ Malonyl ACP transfers only one carbon to the nascent fatty acid during the condensation step. \\\ Only even numbers of malonyl groups are used in fatty-acid biosynthesis. \\ Malonyl ACP is decarboxylated during the condensation step. \\\ Malonyl ACP is used as a temporary catalytic carrier of two carbon units (acetyl ACP). \\\\ What is the most abundant component of chylomicrons? \\\ ApoB-48 \\ Triglyceride \\\ Phospholipid \\\ Cholesterol ester \\\\ The conversion of nascent chylomicrons to mature chylomicrons requires which of the following? \\\ Bile salts \\\ Lipoprotein lipase \\ High-density lipoprotein \\\ Lymphatic system \\\\ The apoproteins B-48 and B-100 are similar with respect to which of the following? \\ They are synthesized from the same gene. \\\ ApoB-48 is a proteolytic product of apoB-100. \\\ Both are found in mature chylomicrons. \\\ Both are found in very-low-density lipoproteins. \\\\ Which of the following is involved in the synthesis of triacylgycerols in adipose tissue? \\ Fatty acids obtained from chylomicrons and VLDL \\\ Glycerol 3-phosphate derived from blood glycerol \\\ 2-Monoacylglycerol as an obligatory intermediate \\\ Lipoprotein lipase to catalyze the formation of ester bonds \\\\ A molecule of palmitic acid, attached to carbon 1 of the glycerol moiety of a triacylglycerol, is ingested and digested. It passes into the blood, is stored in a fat cell, and ultimately is oxidized to carbon dioxide and water in a muscle cell. Choose the molecular complex in the blood in which the palmitate residue is carried from the lumen of the gut to the surface of the gut epithelial cell. \\\ VLDL \\\ Chylomicron \\\ Fatty acid-albumin complex \\ Bile salt micelle \\\\ Which of the following steps in the biosynthesis of cholesterol is the committed rate-limiting step? \\\ The condensation of acetoacetyl-CoA with a molecule of acetyl-CoA to yield -hydroxy- methylglutarylCoA (HMG-CoA) \\ The reduction of HMG-CoA to mevalonate \\\ The conversion of mevalonate to two activated isoprenes \\\ The formation of farnesyl pyrophosphate \\\\ Which one of the following apoproteins acts as a cofactor activator of the enzyme lipoprotein lipase (LPL)? \\\ ApoCIII \\ ApoCII \\\ ApoB-48 \\\ ApoE

\\\\ Which one of the following sequences places the lipoproteins in the order of most dense to least dense? \\\ HDL/VLDL/chylomicrons/LDL \\ HDL/LDL/VLDL/chylomicrons \\\ LDL/chylomicrons/HDL/VLDL \\\ VLDL/chylomicrons/LDL/HDL
\\\\ Which vitamin is derived from cholesterol? \\\ A \\\ B12 \\ D \\\ E

\\\\ The delivery of free fatty acids from lipoproteins into adipose tissue cells will be increased by: \\\ Epinephrine \\\ Diabetes \\\ Glucagon \\ Insulin \\\\ Newly synthesized fatty acids are not immediately degraded. Which of the following best explains this fact? \\\ Tissues that synthesize fatty acids do not contain the enzymes that degrade fatty acids. \\\ High NADPH levels inhibit beta- oxidation. \\\ In the presence of insulin, the key fatty acid degrading enzyme is not induced. \\ Transport of fatty acids into mitochondria is inhibited under conditions in which fatty acids are being synthesized. \\\\ What is true about conversion of acetoacetate to acetone? \\\ might be non-enzymatic \\\ changes one ketone body into another \\\ occurs in the mitochondria \\ all of the above \\\\ Dietary fats (triglycerides) are imported into the body by which tissues? \\\ adipose tissue \\ intestine \\\ liver \\\ pancreas \\\\ In animals, HMG-CoA is synthesized \\ in the cytosol and mitochondrial matrix \\\ on the endoplasmic reticulum and plasma membrane \\\ in the Golgi apparatus primarily \\\ on the endoplasmic reticulum and outer mitochondrial membrane \\\\ Triacylglycerols are cellular metabolic fuels. Which of the following are true about triacylglycerols? \\\ They are polar, hydrophobic molecules. \\\ Most triacylglycerols have three fatty acid groups that are identical. \\\ They should be completely eliminated from the human diet. \\ Triacylglycerols from animals are solids at room temperature while those from plants are liquid. \\\\ How the fats are absorbed by the body during digestion? \\\ They are hydrolyzed by amylase in the mouth. \\ They are emulsified in the small intestine by bile salts. \\\ They are digested in the stomach by stomach acids. \\\ They are transported directly through blood \\\\ Once fats are digested, they are transported by lipoproteins in the blood to the peripheral tissues. What is the role of lipoproteins?

\\\ They act as detergents. \\\ They degrade triacylglycerols into fatty acids and glycerol. \\\ They raise cAMP levels. \\ They keep triacylglycerols in a soluble form \\\\ beta-oxidation of fatty acids occurs on activated fatty acids (fatty acyl CoA). After each set of reactions, the fatty acid chain is reduced by two carbons. These leave in the form of acetyl CoA. Why is this important? \\ Acetyl CoA goes on to further oxidation in the citric acid cycle. \\\ Acetyl CoA recycles back into b oxidation. \\\ Acetyl CoA is further metabolized into acetate. \\\ Acetyl CoA used as a cholesterol precursor. \\\\ Which enzyme catalyzes the production of malonyl-CoA from Acetyl-CoA in the process of fatty acid synthesis? \\\ Acetyl transferase \\ Acetyl-CoA carboxylase \\\ Malate dehydrogenase \\\ Aconitase \\\\ Which statement is correct about absorption of lipids? \\\ Bile salts derived from micelles are excreted from the body with feces. \\\ All products derived from lipids are easily absorbed in the enterocytes \\ TGs re-synthesized in enterocytes are transported in the entherocytes lymph system in the form of chylomicrons. \\\ VLDL provides transport of absorbed lipids \\\\ Which enzyme is hormone-dependent? \\ triacylglycerol lipase \\\ diacylglycerol lipase \\\ monoacylglycerol lipase \\\ pancreatic lipase \\\\ Which is not produced from cholesterol in the body? \\\ Corticosteroids \\\ provitamin D3 \\ Vitamin A \\\ Bile acids \\\\ Which one is correct about citrate? \\\ Is allosteric activator of acyl-CoA \\\ With help of this compound acetyl-CoA is transported from mitochondria into the citosol \\\ Oxaloacetate produced form degradation of the citrate is the source for CO2 and NADPH which are essential for fatty acid synthesis. \\ All answers are correct \\\\ What is characteristic for blood plasma lipoproteins? \\\ TGs resynthesized in the enterocytes are transported in the content of VLDL. \\\ TGs resynthesized in the liver are transported in the content of chylomicrons \\\ HDL is characterized with high content of HDL \\ LDL involves in the transportation process of cholesterol and its esthers \\\\ What is the function of LCAT (Lecitin cholesterol acyl trasnferase)? \\\ It catalyzes reaction between phospholipids and free cholesterol on the surface of HDL \\\ Lisolecitin and esterified cholesterol are produced after LCAT action \\\ esterified cholesterol which is produced with LCAT reaction finally utilizes in the liver \\ All listed concepts are correct

\\\\ Which compound does not take place in the process of cholesterol biosynthesis? \\\ Beta-hydroxy beta methyl glutaryl CoA \\\ Mevalonate \\ Malonyl-CoA \\\ Squalen \\\\ What is characteristic for cholesterol metabolism? \\\ Synthesis of cholesterol takes place in the mitochondria \\\ Regulatory enzyme of the biosynthesis is thyolase \\ LDL is the richest fraction with cholesterol \\\ LDL is the poorest fraction with cholesterol \\\\ Which compounds are produced after action of pancreatic lipase? \\\ Triacylglycerol \\ Free fatty acids and 2 mono acyl glycerol \\\ Glycerol, fatty acids and cholin \\\ Fatty acids and cholesterol \\\\ Which is not characteristic for ketone bodies metabolism? \\\ Ketogenesis occurs in liver mitochondria \\\ Ketogenesis intensifies because of inhancing of lipolysis \\ Oxidation of ketone bodies goes in liver cells citosol \\\ Hiperketonemia is characteristic for diabetes mellitus \\\\ How the intracellular lipolysis is regulated? \\ The regulatory enzyme of the process is triacylglycerol lipase \\\ The regulatory enzyme of the process is diacylglycerol lipase \\\ triacylglycerol lipase is inhibited by adrenalin, glucagon, adrenocorticotropic hormone \\\ insulin activates the process \\\\ Which hormone inhibits degradation of triacylglycerols (lipolysis)? \\\ Adrenalin \\\ Glucagon \\\ Adrenocorticotropic hormone \\ Insulin \\\\ Which one is not lipid digestive enzyme? \\\ Phospholipase \\\ Cholesterol esterase \\ Peptidase \\\ Lipase \\\\ Which is not correct concerning acetyl-CoA carboxylase? \\\ Its prostethic group is biotin \\\ Its allosteric activator is citrate \\\ Its synthesis inductor is insulin \\ Takes part in TCA cycle \\\\ What is not bile function? \\\ Neutralization of chymus acid reaction \\\ Taking part in the process of absorption of lipid soluble vitamins \\\ Excretion of toxins from the body \\ Digestion of protein \\\\ Activation of fatty acids goes with: \\ CoA

\\\ CTP \\\ Carnitine \\\ UTP \\\\ Activation of fatty acids catalyzes with enzyme: \\\ Carnitin palmitoil-transferase II \\ Acyl-CoA synthetase \\\ Acyl-CoA dehydrogenase \\\ Translocase \\\\ Which compound transfers acyl-CoA from cytosol to mitochondria? \\ carnitine \\\ ATP \\\ GTP \\\ CoA \\\\ How Acyl CoA is transported into the mitochondria? \\\ With help of active transport \\ Carnitine is essential for entering of Acyl CoA in the mitochondria \\\ Carnitin palmitoil-transferase II catalyzes this reaction \\\ Translocase is not involved in the process \\\\ Acyl-CoA ----- enoyl-CoA the enzyme of the reaction is: \\ Acyl-CoA dehydrogenase \\\ Succinate dehydrogenase \\\ Acyl-CoA synthetase \\\ Beta-hydroxyacyl CoA dehydrogenase \\\\ How many acetyl CoA is produced after beta-oxidation of palmitate? \\\ 6 \\ 8 \\\ 9 \\\ 12 \\\\ how many beta-spirals are required for beta-oxidation of palmitate? \\\ 6 \\ 7 \\\ 10 \\\ 12 \\\\ how many macroergic bonds of ATP are required for acyl-CoA synthetase reaction? \\\ 1 \\ 2 \\\ 3 \\\ 6 \\\\ Fatty acids biosynthesis starts from: \\\ Succinyl-CoA \\ Acetyl-CoA \\\ Acetoacetate \\\ Succinate \\\\ Which compound participates in the process of fatty acids elongation with 2 carbons? \\\ Lactate \\\ Malate \\\ Propionate \\ Malonyl-CoA

\\\\ How many NADPH are added to process of palmitate biosynthesis from pentose phosphate pathway? \\\ 3 \\ 6 \\\ 7 \\\ 8 \\\\ Which decarboxylating dehydrogenase is involved in the process of palmitate biosynthesis? \\\ Pyruvate dehydrogenase \\\ Isocytrate dehydrogenase \\\ Alpha ketoglutarate dehydrogenase \\ Malic enzyme (cytosolic malate dehydrogenase) \\\\ Which lipoprotein is source of Apo E and Apo C for maturing the chylomicrons? \\\ LDL \\\ VLDL \\ HDL \\\ IDL \\\\ Which apolipoprotein exist in the immature chylomicron? \\\ ApoC and Apo E \\ Apo B-48 \\\ ApoC-1 and Apo-D \\\ ApoB-100 \\\\ Which apolipoprotein exist in the immature VLDL? \\ Apo B-100 \\\ ApoA \\\ ApoE \\\ ApoD \\\\ VLDL maturation occurs after addition of : \\\ ApoB-100 and ApoC \\ ApoC and ApoE \\\ ApoE and ApoA \\\ ApoC-I and ApoB-48 \\\\ Which lipoprotein is precursor of IDL? \\\ Chylomicron \\ VLDL \\\ HDL \\\ LDL \\\\ HDL is: \\\ Chylomicron remnants \\\ Bad lipoprotein \\ Lipoprotein, to which LCAT is attached \\\ TAG-rich lipoprotein \\\\ Which is the inductor of acetyl-CoA carboxylase synthesis? \\\ Glucagon \\\ Adrenalin \\\ Adrenopcorticotropic hormone \\ Insulin \\\\ in glycolytic pathway glycerol is involved by form of: \\\ Pyruvate

\\ Dihydroxyaceton phosphate \\\ Lactate \\\ Phospho enolpyruvate \\\\ How glycerol is activated? \\\ By acetylation \\\ By uridylation \\\ By methylation \\ By phosphorylation \\\\ Which enzyme activates glycerol? \\\ Glyceraldehyde phosphate dehydrogenase \\\ Thiolase \\ Glycerol kinase \\\ aldolase \\\\ What is the common intermediate compound of cholesterol biosynthesis and ketogenesis? \\\ Succinyl-CoA \\ HMG-CoA \\\ Alpha ketoglutarate \\\ Chenodeoxycholic acid \\\\ The starting compound of cholesterol biosynthesis is: \\\ acyl-CoA \\ acetyl-CoA \\\ acetoacetate \\\ cholic acid \\\\ Which coenzyme is involved in cholesterol biosynthesis process? \\\ NAD+ \\\ FAD \\ NADPH \\\ TPP \\\\ Which is not intermediate compound of cholesterol? \\ cholic acid \\\ isopentenylpyrophosphate \\\ pharnesipyrophosphate \\\ mevalonate \\\\ How insulin acts on lipid metabolism? \\\ intracellular cAMP concentration increases in adipose tissue \\\ triacylglycerol lipase activates \\\ activates intracellular lipolysis \\ activates lipid storage process \\\\ Which enzyme degrades chylomicrons and VLDL triacylglycerols? \\\ HMG-CoA reductase \\\ lipoprotein lipase \\ TG-lipase \\\ phospholipase \\\\ What is the allosteric inhibitor of acetyl-CoA carboxylase? \\\ acetyl-CoA \\ palmitoyl-CoA \\\ Malonyl-CoA \\\ acetoacetate

\\\\ In duodenum lipid emulsification occurs by action of: \\\ HCl \\ bile \\\ intestinal juice \\\ gastric juice \\\\ Which statement is not characteristic for ketone body metabolism? \\\ Their utilization occurs in liver \\\ NADH/NAD+ ratio in mitochondria determines the ratio of beta-hydroxybutirate and acetoacetate \\ In case of high ratio of NAD/NADH+ concentration of beta-hydroxybutirate increases in liver \\\ hyperketonemia causes ketoacidoses. \\\\ Which enzymes low activity causes obesity? \\ triacylglycerol lipase \\\ diacylglycerol lipase \\\ monoacylglycerol lipase \\\ phospho lipase \\\\ Which compound takes part in fatty acid elongation process? \\ acetyl-CoA \\\ Lactate \\\ Malonyl-CoA \\\ Malate \\\\ What is the substrate of lipoprotein lipase? \\ TG existing in VLDL \\\ TG existing in LDL \\\ TG existing in HDL \\\ TG existing in membranes \\\\ Which lipoprotein inhibits cholesterol biosynthesis? \\\ VLDL \\\ HDL \\\ IDL \\ LDL \\\\ To which lipoprotein is LCAT (lecithin: cholesterol: acyl transferase) bound? \\\ VLDL \\ HDL \\\ IDL \\\ LDL \\\\ free fatty acids are produced after hydrolysis of: \\\ Histons \\ triacylglycerols \\\ chromoproteins \\\ phosphoproteins \\\\ What is the role of citrate in lipid metabolism? \\\ provides transport of acetyl-CoA from cytosol into mitochondria \\\ inhibits acetyl-CoA carboxylase \\ oxaloacetate produced from citrate provides the cells with NADPH \\\ inhibits malic-enzyme. \\\\ allosteric activator of acetyl-CoA-carboxilase is: \\ citrate

\\\ palmitoyl-CoA \\\ acetyl-CoA \\\ Malonyl-CoA \\\\ Which is not involved in the process of triacylglycerols? \\\ glycerol-3-phosphate \\\ phosphatidic acid \\\ lisophophatidic acid \\ acetoacetate \\\\ How insulin acts on lipid metabolism? \\ inhibits intracellular lipolysis \\\ prevents triacylglycerol storage in adipose tissue \\\ causes repression of synthesis of acetyl-CoA carboxylase \\\ prevents glucose conversion into glucose \\\\ Which lipoprotein transports dietary cholesterol to the liver? \\ chylomicrons \\\ LDL \\\ HDL \\\ VLDL \\\\ What is wrong about long-chain fatty acids? \\\ Fatty acids are hydrophobic compounds \\\ they have toxic features \\ fatty acid activation need GTP energy \\\ palmitate and sterate belong to saturated long-chain fatty acids. \\\\ What compound is necessary for transporting long-chain fatty acids from cytosol into mitochondria? \\ transporter carnitine \\\ Na+K+ATP-ase \\\ G-protein \\\ UTP \\\\ Which compound is received on the last stage of beta-oxidation of fatty acids? \\ propionyl-CoA \\\ Acetyl-CoA \\\ Butiril-CoA \\\ Succinyl-CoA \\\\ What statement is not correct concerning acetyl-CoA carboxilase? \\\ catalyses rate limiting step of fatty acid synthesis \\ activates by palmitoil-CoA \\\ requires biotin \\\ inductor of the synthesis is insulin \\\\ In which metabolic pathway acetyl-CoA is not used? \\\ in the oxidation pathway of acetyl-group for energy generation \\\ in the process of ketone bodies synthesis \\\ in the process of long-chain fatty acid synthesis \\ in the gluconeogenetic process for increasing glucose concentration \\\\ acetyl-CoA + HCO3-+ ATP --------- Malonyl-CoA+H2O+ADP+Pi this reaction is: \\ rate-limiting step of fatty acid synthesis \\\ intermediate step in the process of TAG synthesis \\\ the precursor reaction for TCA cycle \\\ example of anaplerotic reaction

\\\\ How the brain adapts for starvation? \\\ with anaerobic oxidation of glucose \\\ with beta-oxidation of fatty acids \\ with ketone bodies oxidation \\\ with glycerol oxidation \\\\ How pharmacologic drugs statins act as hypocholesterolemic agents? \\ inhibit HMG-CoA reductase \\\ activate HMG-CoA reductase \\\ inhibit acetoacetyl-CoA thiolase \\\ activate mevalonate kinase \\\\ Which statement is not correct concerning comparison of lipid and carbohydrate metabolism? \\ after oxidation of triacylglycerols less ATPs are synthesized compared to glycogen \\\ triacylglycerols are stored without water and glycogen - with twice more water . \\\ fats are not used as energy source in anaerobic conditions \\\ fats are slowly mobilized than glycogen \\\\ Which of the following is characteristic for insulin resistance during diabetes mellitus type 2 ? \\\ obesity and insulin resistance does nor correlate \\\ tissues respond to insulin with hypersignaling \\\ concentration of free fatty acids are declined in the blood \\ glucose utilization in muscles is declined \\\\ Which clinical signs are not characteristic for insulin resistance? \\\ increased glucose level in the blood \\\ increased insulin level in the blood \\\ obesity \\ decreased free fatty acid concentration in the blood \\\\ What is characteristic for starvation phase? \\\ storage of glycogen and triacylglycerols \\ utilization of ketone bodies by the brain for energetical issues \\ inhibition of gluconeogenesis in liver \\\ decreasing concentration of free fatty acids and glycerol in the blood \\\\ After synthesis of glycerol-3-phosphate from glycerol makes this compound available for glycolitic and gluconeogenetic pathways. Which enzyme provides phosphorylation of glycerol? \\ glycerol kinase \\\ glycerol-3-phosphate dehydrogenase \\\ glyceraldehyde phosphate dehydrogenase \\\ triose phosphate dehydrogenase

Hormones
\\\\ What process of maturation of insulin from its precursor (preproinsulin) involves: \\\ acetylation. \\\ phosphorylation. \\ proteolysis. \\\ reduction. \\\\ What is the effect of elevated insulin level in the blood? \\\ inhibion of glucose uptake by the liver. \\\ inhibition of glycogen synthesis in the liver and muscle. \\\ stimulation of glycogen breakdown in liver. \\ stimulation of synthesis of fatty acids and triacylglycerols in the liver. \\\\ What is the effect of elevated insulin level in the blood? \\\ fatty acid mobilization in adipose tissue. \\\ gluconeogenesis in liver. \\\ glycogen breakdown in muscle. \\ glycogen synthesis in liver.

\\\\ What process is activated by anabolic hormone insulin? \\\ Degradation of proteins with formation of amino acids \\\ Convertion of glycogen to glucose \\\ The conversion of amino acids to glucose \\ The conversion of glucose to fatty acids and triacylglycerol \\\\ Which of the following is not called counter regulatory (they counter the effects of insulin) hormone? \\\ Glucagon \\\ Norepinephrine \\\ Cortisol \\ Growth factor \\\\ In which of the following processes the hormone insulin is not participating? \\\ The synthesis of fatty acids from glucose \\\ The synthesis of triacylglycerols in liver and adipose tissue \\ The mobilization of amino acids from proteins for gluconeogenesis \\\ The synthesis of proteins \\\\ You would expect that the release of insulin would be the greatest, when: \\\ digging ditches for 4 hours \\\ running from a gorilla \\ eating a high carbohydrate \\\ eating a high protein meal \\\\ You would expect the release of glucagon to be least \\\ When eating a high protein meal \\ When eating a high carbohydrate meal \\\ When pushing a car for 20 miles \\\ During hypoglycemia \\\\ As a group, you would expect the stress hormones to \\\ Increase the synthesis of fatty acids in the liver \\\ Increase triacylglycerol synthesis in liver and adipose tissue \\ Increase the utilization of muscle protein for glucose synthesis \\\ Increase glycogen synthesis \\\\ Which one of the following events regarding the synthesis of insulin is true? \\\ Insulin is not synthesized as a preprohormone, its synthesized like steroid hormones

\\\ Cleavage of the signal peptide in the endoplasmic reticulum converts the active insulin to inactive prohormone form. \\ Formation of disulfide bonds and cleavage of the C-peptide and a few amino acids by proteases converts the prohormone into insulin \\\ Insulin precipitates with protamine in the storage vesicles of beta-cells of the pancreas \\\\ Which of the following is true about the release of insulin from beta-cells? \\\ Insulin secretion increases during illness because of epinephrine binding to receptors \\\ Insulin secretion decreases following the initiation of exercise because of epinephrine binding to receptors \\\ Insulin secretion impaires following a high protein diet in response to increased concentrations of amino acids \\ Insulin secretion increases following a high carbohydrate meal in response to increased concentrations of glucose \\\\ Which of the following is true concerning the release of glucagon from alpha-cells? \\\ Insulin will bind to alpha-cells and increase the release of glucagon \\\ A high carbohydrate meal will increase the release of glucagon \\\ Hyperglycemia will increase the release of glucagon \\ Trauma and other types of stress will increase the release of glucagon \\\\ When insulin increases, it binds to insulin receptors on muscle cells. What is the result of this? \\ The receptor changes conformation and autophosphorylation of the insulin receptor occurs \\\ Before autophosphorylation, the insulin receptor phosphorylates seryl residues on the IRS protein \\\ A chain of reactions occur that eventually activate cAMP \\\ cAMP initiates a sequence of events that results in Glut-1 moving from storage vesicles to the membrane so that there is an increase in glucose transport \\\\ When a fasting person eats a high carbohydrate or mixed meal, the concentration of glucagon may decreases or remains the same, but the second messenger system of glucagon is wiped out. How this process is explained? \\ Insulin activates cAMP phosphodiesterase, an enzyme that converts cAMP into AMP \\\ Insulin activates protein kinases that phosphorylates proteins that were dephosphorylated by protein kinase A \\\ Proteins like liver phosphofructokinase-2/fructose-2,6-bisphosphatase are phosphorylated by protein kinases \\\ Proteins like pyruvate kinase are phosphorylated by protein phosphatase \\\\ What process will promote when glucagon binds to its receptor on the liver membrane? \\\ A change in conformation of the glucagon receptor results in binding to Gi protein and release of bound GTP \\\ The binding of GTP to Gs protein causes association of the alpha subunit with beta-gamma subunit \\\ cAMP will convert into AMP by phosphodiesterase. \\ Active protein kinase A, activated by cAMP, will phosphorylate other proteins and the activity of regulatory enzymes will be changed \\\\ Your patient has an insulinoma and suffers from fasting hypoglycemia. Which of the following would help explain why the abnormally high insulin levels would cause fasting hypoglycemia? \\\ Insulin is signaling cells to degrade liver glycogen, thus increase the blood glucose \\ Insulin is inhibiting gluconeogenesis so blood glucose cannot be renewed from this source \\\ Insulin is activating glycolysis and fatty acid synthesis in muscle so these sources of ATP are a lot. \\\ Insulin is activating the release of free fatty acids from adipose so even more glucose is needed to maintain the ATP of most cell types \\\\ Your patient has an insulinoma and suffers from fasting hypoglycemia. Which of the following statements would be true? \\\ Hypoglycemia is caused by the release of epinephrine and glucagon \\\ Both glucagon and epinephrine decrease the concentration of glucose in the blood.

\\\ Insulin will activate the release of glucagon from alpha-cells \\ Insulin is released in fasting state also. \\\\ Which of the following statements about hormonal levels during different states is true? \\\ During the time you are eating a high carbohydrate mixed meal, the insulin/glucagon ratio will decrease \\\ When passing from the fed to fasting state, insulin and glucagon usually decrease \\\ When playing basketball, epinephrine is usually low and insulin is high \\ After running for 20 miles, epinephrine, glucagon and cortisol are high and insulin is low \\\\ When the insulin/glucagon ratio falls (when you begin to exercise, run, etc.) which of the following will occur? \\\ Glycogen synthase will be activated by phosphorylation by protein kinase A \\\ Glycogen phosphorylase will be deactivated by phosphorylation by phosphorylase kinase \\ Phosphorylase kinase will be activated by phosphorylation by protein kinase A \\\ Increased cAMP will bind to and combine the regulatory and catalytic subunits of protein kinase A \\\\ insulin activates protein phosphatase, what is the result of action of protein phosphatase? \\ activates glycogen synthase by dephosphorylation \\\ Inactivates phosphorylase kinase by dephosphorylation \\\ Inactivates glycogen phosphorylase by phosphorylation \\\ Activates acetyl CoA carboxylase by phosphorylation \\\\ Which of the following statements best describes glucagon? \\\ It acts as an anabolic hormone. \\ It acts primarily on the liver and adipose tissue. \\\ Its concentration in the blood increases after a high-carbohydrate meal. \\\ Its concentration increases in the blood when insulin levels increase. \\\\ Which of the following is most likely to occur in a normal individual after ingesting a high-carbohydrate meal? \\ Only insulin level increases. \\\ Only glucagon level increases. \\\ Both insulin and glucagon levels decrease. \\\ Both insulin and glucagon levels increase. \\\\ After binding of glucagon to a liver cell, what process is activated? \\\ G-protein binds to ATP \\ Adenylate cyclase is activated \\\ Protein phosphatase is activated \\\ Cyclic-AMP is decreased \\\\ After a 24 hour fast, the maintenance of blood glucose is largely a result of the change in the insulin to glucagon ratio. Which of the following help to explain this mechanism? \\\ More free fatty acids and glycerol are stored in adipose tissue \\\ More malonyl-CoA is produced from free fatty acids in the liver \\ Acetyl CoA activates pyruvate carboxylase \\\ Oxaloacetate is used as the principal source of energy to drive gluconeogenesis \\\\ All of the following actions of glucagon are true EXCEPT \\\ Glucagon will inhibit glycogen synthesis and activate glycogenolysis \\\ Glucagon will activate fatty acid mobilization (release) in adipose tissue \\ Glucagon will activate triacylglycerol synthesis in liver and adipose tissue \\\ Glucagon will remove amino acids for gluconeogenesis and thus increase the mobilization of amino acids from proteins \\\\ When the glucagon concentration outside a liver cell is decreased suddenly, there is rapid change in the activation of many of the pathways influenced by glucagon. Which of the following is not responsible for the

rapid termination of signal? \\\ Without glucagon bound, receptors can no longer activate Gs protein \\ The G-beta-gama subunit hydrolyzes GTP and is no longer active \\\ cAMP phosphodiesterase removes cAMP from the cell \\\ Protein phosphatases remove phosphate groups and cause some enzymes to be more active \\\\ When epinephrine is bound to the following receptors, which one activates the IP3 pathway? \\ Alpha-1 adrenergic receptor \\\ Beta-1 adrenergic receptor \\\ Beta-2 adrenergic receptor \\\ Beta-3 adrenergic receptor \\\\ Assume that you find a patient with a glucagon-secreting pancreatic tumor (glucagonoma). Which one of the following is most likely to result from hyperglucagonemia? \\ Hyperglycemia \\\ Increased muscle protein synthesis. \\\ Increased liver glycolytic rate \\\ Increased glycogenesis \\\\ Assume that you find a patient with a glucagon-secreting pancreatic tumor (glucagonoma). Which one of the following is most likely to result from hyperglucagonemia? \\ increased lipolysis \\\ Increased muscle protein synthesis. \\\ Increased liver glycolytic rate \\\ Increased glycogenesis \\\\ A typical characteristic of hormones that use the second messenger mechanism is that: \\ they can not pass through the plasma membrane \\\ the receptor is located in the cytosol \\\ the receptor is not necessary \\\ the receptor binds to the hormone to allow it to enter the cell \\\\ Receptors for steroid hormones can be found: \\\ in the plasma membrane \\ in the cytosol \\\ in the mitochondria \\\ in the ribosomes \\\\ The binding of insulin to the insulin receptor triggers: \\\ the activation of adenyl ciclase \\\ the activation of phospholipase C \\\ a decrease of GLUT4 in the plasma membrane \\ tyrosine kinase activity \\\\ Which of the of the following hormones use cAMP as second messenger? \\\ Insulin \\\ Thyroxin \\\ Progesterone \\ Glucagon \\\\ Hormones that act through Phospholipase C activation, in fact are using three second messengers: \\\ cAMP, , Ca++ and Diacylglycerol \\\ cAMP, Diacylglycerol and Inositol 1,4,5 triphosphate \\\ Ca++, Diacylglycerol and ceramide \\ Ca++, Diacylglycerol and Inositol 1.4.5 triphosphate

\\\\ What do insulin, glucagon, and epinephrine have in common? \\\ They increase blood glucose levels. \\ They act through signal transduction. \\\ They are all proteins. \\\ They are all released by the pancreas.

AMINO ACIDS
\\\\ In amino acid catabolism, what is the first reaction for many amino acids? \\\ decarboxylation requiring thiamine pyrophosphate (TPP). \\\ hydroxylation requiring NADPH and O2. \\\ oxidative deamination requiring NAD+. \\ transamination requiring pyridoxal phosphate (PLP). \\\\ What is the source of the coenzyme required for all transaminases? \\\ niacin. \\\ pyridoxine (vitamin B6) \\\ riboflavin. \\\ ciancobalamin (vitamin B12). \\\\ Which coenzyme is involved in a transaminase reaction? \\\ lipoic acid. \\\ nicotinamide adenine dinucleotide phosphate (NADP+). \\ pyridoxal phosphate (PLP). \\\ thiamine pyrophosphate (TPP). \\\\ Transamination from alanine to alpha-ketoglutarate requires the coenzyme: \\\ NADH. \\\ No coenzyme is involved. \\ pyridoxal phosphate (PLP). \\\ thiamine pyrophosphate (TPP). \\\\ Which statement is correct for the conversion of glutamate to an alpha-ketoacid and NH4+? \\\ does not require any cofactors. \\\ is a reductive deamination. \\\ is accompanied by ATP hydrolysis catalyzed by the same enzyme. \\ is catalyzed by glutamate dehydrogenase. \\\\ Primarily in which tissues urea synthesis takes place in mammals? \\\ brain. \\\ kidney. \\ liver. \\\ small intestine. \\\\ Which substance is involved in the production of urea from NH4+ via the urea cycle? \\\ mevalonate \\\ TTP \\ Carbamoyl phosphate \\\ Malate \\\\ Which of these directly donates a nitrogen atom for the formation of urea during the urea cycle? \\\ Adenine \\ Aspartate \\\ Creatine \\\ Ornithine \\\\ Which of these amino acids are both ketogenic and glucogenic? \\ Isoleucine \\\ Valine \\\ Histidine \\\ Arginine

\\\\ The nitrogens in urea are directly derived from which of the following compounds? \\\ Ornithine and carbamoyl phosphate \\\ Ornithine and aspartate \\\ Ornithine and glutamate \\ Carbamoyl phosphate and aspartate \\\\ Which one of the following enzymes can fix ammonia into an organic molecule? \\\ Alanine-pyruvate aminotransferase \\ Glutamate dehydrogenase \\\ Arginase \\\ Argininosuccinate synthetase \\\\ which of the following is the major regulated step of urea cycle? \\ Carbamoyl phosphate synthetase 1 \\\ Ornithine transcarbamoylase \\\ Argininosuccinate synthetase \\\ Argininosuccinate lyase \\\\ Which of the following enzyme reactions takes place during the synthesis of urea from ammonium ion and glutamate? \\\ Carbamoyl phosphate + citrulline = ornithine \\ Aspartate + citrulline + ATP = argininosuccinate + AMP + PPi \\\ CO2 + NH4+ + 2 ADP = carbamoyl phosphate + 2 ATP \\\ Argininosuccinate = arginine + urea \\\\ Which of the following would be expected to increase the activity of the urea cycle: \\\ Antibiotics \\\ Switching to a high carbohydrate diet \\ An increase in gluconeogenesis \\\ High insulin concentration \\\\ Which of the following is an essential amino acid? \\\ Alanine \\\ Glycine \\ Valine \\\ Aspartate \\\\ The urea cycle is regulated by: \\\ Acetyl CoA \\\ Malonyl CoA \\ N-acetylglutamate \\\ Cytoplasmic carbamoyl phosphate \\\\ By which form the carbon structure of this amino acid can be converted in a single enzyme step into an interemediate in either the TCA cycle or the urea cycle? \\\ Arginine \\\ Citrulline \\ Aspartate \\\ Glutamine \\\\ Which of the following is an essential amino acid: \\\ Alanine \\ Tryptophan \\\ Glutamine \\\ Asparagine \\\\ Which of the following amino acids is ketogenic?

\\ Leucine \\\ Alanine \\\ Glutamate \\\ Phenylalanine \\\\ Which of the following is characteristic for aminotransferases? \\\ usually require alpha-ketoglutarate or glutamine as one of the reacting pair. \\\ catalyze irreversible reactions. \\ require pyridoxal phosphate as an essential cofactor for the reaction. \\\ are not able to catalyze transamination reactions with essential amino acids. \\\\ Which of the following is not true of the reaction catalyzed by glutamate dehydrogenase? \\ It is similar to transamination in that it involves the coenzyme pyridoxal phosphate (PLP). \\\ NH4+ is produced. \\\ The enzyme can use either NAD+ or NADP+ as a cofactor. \\\ The enzyme is glutamate-specific, but the reaction is involved in oxidizing other amino acids. \\\\ Which of the following statements about glutamate is not true: \\\ It can be synthesized in the transaminase reaction using alanine as a substrate \\\ It can be synthesized by the glutamate dehydrogenase reaction using alpha-ketoglutarate and free ammonium ion as substrates \\\ It can transfer it's amino group to oxaloacetate in a one step reaction \\ It can transfer it's amino group to citrulline in a one step reaction

Hemoglobin, myoglobin
\\\\ When oxygen binds to a heme-containing protein, by what of the following two coordination bonds of Fe2+ are occupied? \\\ one O atom and one amino acid atom. \\ one O2 molecule and one amino acid atom. \\\ one O2 molecule and one heme atom. \\\ two O2 molecules. \\\\ What curve is characteristic for oxygen binding to myoglobin? \\ hyperbolic. \\\ linear with a negative slope. \\\ linear with a positive slope. \\\ sigmoidal. \\\\ What is characteristic for myoglobin and the subunits of hemoglobin? \\\ They have no obvious structural relationship. \\\ They have very similar primary and tertiary structures. \\\ They have very similar primary structures, but different tertiary structures. \\ They have very similar tertiary structures, but different primary structures. \\\\ by what is triggered the transition from T state to R state (low to high affinity) in hemoglobin? \\\ Fe2+ binding. \\\ heme binding. \\ oxygen binding. \\\ subunit association. \\\\ The amino acid substitution of Val for Glu in Hemoglobin S results in aggregation of the protein. What type of interactions between molecules is the main reason of the aggregation?. \\\ covalent \\\ disulfide \\ hydrophobic \\\ ionic \\\\ The fundamental cause of sickle-cell disease is a change in the structure of: \\\ blood. \\\ capillaries. \\ hemoglobin. \\\ the heart.

\\\\ What chains are found in in HBA2 as well as in HBF? \\ hemoglobin alpha-chains \\\ hemoglobin beta-chains \\\ hemoglobin gama-chains \\\ hemoglobin sigma-chains \\\\ What statement is correct for 2,3 bisphosphoglycerate (2,3 BPG)? \\\ is absent in the normal erythrocytes \\\ is a homotropic effector for hemoglobin \\\ binds more tightly to HbF than to HbA \\ synthesis increases when hemoglobins T-R equilibrium is shifted in favor of the T state. \\\\ What factor increases the affinity of hemoglobin to oxygen? \\ high concentration of oxygen \\\ high concentration 2,3 BPG \\\ low pH \\\ high concentration of CO2 \\\\ What is the composition of HbA2? \\\ alpha2-epsylon2

\\ alpha2-beta2 \\\ alpha2-sigma2 \\\ alpha2-gama2 \\\\ How does the high concentration of 2,3- BPG influence on hemoglobin affinity to oxygen? \\\ no influence \\\ increases affinity \\ decreases affinity \\\ change of affinity under 2,3, BPG is depend on the state of iron in hemoglobin. \\\\ Which of the following is correct concerning oxygen binding to hemoglobin? \\ the process goes according to the positive cooperativety \\\ Deoxyhemoglobin is R form of hemoglobin \\\ Hemoglobin binds only with 1 molecule of oxygen \\\ Hemoglobin saturation curve has hyperbolic form. \\\\ Which of the following is not correct about oxygenation of hemoglobin? \\\ Oxidation state of Fe ion is the same (Fe2+) in both forms - HbO2 and Hb \\\ in deoxihemoglobin ferous ion is 0.4-0.6 A outside of the plate \\\ T form of hemoglobin is deoxihemoglobin and R form oxihemoglobin \\ HbA has higher affinity to O2 compared to myoglobin \\\\ Which of the following is wrong about hemoglobin? \\\ Hemoglobin has tetrameric structure \\\ hemoglobin is oxygen-binding protein and participates in oxygen transport \\ hemoglobin saturation curve has hyperbolic form \\ hemoglobin saturation curve has sigmoidal form \\\\ Which is correct concerning Borh effect? \\\ pH has not important influence on T--R equilibrium. \\ pH decrease provides release of oxygen from HbO2 \\\ pH influences on only T conformation of Hb. \\\ pH decrease inhibits release of oxygen from HbO2 \\\\ Which one is not correct about mechanism of cooperative binding of oxygen to hemoglobin? \\\ Binding of first oxygen to T conformation goes with lower rate. \\\ Binding of oxygen to heme provides shifting of His F8 to porphirin ring. \\ Binding of oxygen to heme causes formation of stabilizing salt bonds of T conformation. \\\ Oxygenation-deoxygenation of hemoglobin is accompanied by conformational changes in the molecule. \\\\ Which is the wrong statement about 2,3-BPG: \\\ 2,3-BPG is the modulator of T-- R equilibrium \\\ high concentration of 2,3-BPG decreases the affinity of Hb to oxygen \\ low concentration of 2,3-BPG decreases the affinity of Hb to oxygen \\\ 2,3-BPG connects to deoxyhemoglobin. \\\\ Which is not correct about Heme? \\\ contains protoporphirin IX ring \\\ It is O2 containing site \\ It is apoprotein \\\ There is Fe2+ ion in the centre of the molecule. \\\\ Which of the hemoglobin derivatives contain Fe3+? \\\ HbCO \\ HbM \\\ HbO2 \\\ HbS

\\\\ Which is wrong about myoglobin? \\\ myoglobin is globular protein \\\ contains polipeptide chain and one heme group \\ myoglobin has low affinity to oxygen \\\ heme group is in hydrophobic pocket of the molecule \\\\ Which of the following is correct concerning deoxihemoglobin? \\\ It is R form of Hb \\ It is weaker acid compared to oxihemoglobin \\\ The 6th coordinative bond in the molecule is occupied \\\ iron atom is in the state Fe3+ \\\\ Which of the following is correct concerning cooperativity? \\ there exist positive and negative cooperativity \\\ cooperativity is characterised for Mb molecule \\\ characterises decreasing affinity of Hb to oxygen \\\ Because of this feature saturation curve of Hb has hyperbolic character . \\\\ Increased concentration of H+ ions in the tissue provide: \\\ increase affinity of deoxyHb to O2 \\\ convertion of T form of Hb into R form \\ dissociation of O2 from oxyHb \\\ conversion of oxyHb ( not deoxyHb) into Mb

Você também pode gostar